Download as pdf or txt
Download as pdf or txt
You are on page 1of 67

Chapter 10

Separate and Consolidated Financial Statements -


Stock Acquisition
I. Introduction
An investment in outstanding common stock creates a parent-subsidiary relationship, the
purchasing entity (parent company) and the entity acquired (subsidiary) continue to
function as separate entities and to maintain their accounting records on a separate legal
basis. Separate parent company and subsidiary financial statements are converted into
consolidated financial statements that reflect the financial position and the results of
operations of the combined entity.

Consolidated Financial Statements, states, ". . . consolidated statements are more


meaningful than separate statements and are usually necessary for a fair presentation
when one of the companies in the group directly or indirectly has a controlling financial
interest in the other companies". All majority-owned subsidiaries - all companies in which a
parent has a controlling financial interest through direct or indirect ownership of a
majority voting interest - should be consolidated.

//. The following summaries of PAS 27 - Separate Financial Statements (2017)


and PFRS 10-Consolidated Financial Statements (2017):

1. Objectives:

• PAS 27: It has the objective of setting standards to be applied in accounting


for investments in subsidiaries, jointly ventures, and associates when an entity
elects, or is required by local regulations, to present separate (non-
consolidated) financial statements.

• PFRS 10: It is to establish principles for the presentation and preparation of


consolidated financial statements when an entity controls one or more other
entities.

2. Key Definitions (PAS 27 and PFRS 10):
Consolidated Financial Statements - the financial statements of a group in which the
assets, liabilities, equity, income, expenses and cash flows of the parent and its
subsidiaries are presented as those of a single economic entity
Separate Financial Statements - Financial statements presented by a parent (i.e. an
investor with control of a subsidiary), an investor with joint control of, or significant
influence over, an investee, in which the investments are accounted for at cost or in
accordance with PAS 39 or PFRS 9 Financial Instruments (effective 2015)

Control of an Investee - an investor controls an investee when the investor is


exposed, or has rights, to variable returns from its involvement with the investee
and has the ability to affect those returns through its power over the investee

Investment entity - An entity that:


• obtains funds from one or more investors for the purpose of providing
those investor(s) with investment management services
• commits to its investor(s) that its business purpose is to invest funds
solely for returns from capita! appreciation, investment income, or both, and
measures and evaluates the performance of substantially all of its
investments on a fair value basis.

3. Preparation of consolidated financial statements


A parent prepares consolidated financial statements using uniform accounting
policies for like transactions and other events in similar circumstances.

However, a parent need not present consolidated financial statements if it meets all
of the following conditions:

• it is a wholly-owned subsidiary or is a partially-owned subsidiary of another


entity and its other owners, including those not otherwise entitled to vote,
have been informed about, and do not object to, the parent not presenting
consolidated financial statements
• its debt or equity instruments are not traded in a public market (a domestic
or foreign stock exchange or an over-the-counter market, including local
and regional markets)
• it did not file, nor is it in the process-' of filing, its financial statements with a
securities commission or other regulatory organization for the purpose of issuing any
class of instruments in a public market, and
• its ultimate or any intermediate parent of the parent produces consolidated
financial statements available for public use that comply with PFRSs.

Investment entities are prohibited from consolidating particular subsidiaries.

Furthermore, post-employment benefit plans or other long-term employee benefit


plans to which PAS 19 Employee Benefits applies are not required to apply the
requirements of PFRS 10.

4. Consolidation procedures
Consolidated financial statements:
• combine like items of assets, liabilities, equity, income, expenses and cash
flows of the parent with those of its subsidiaries
• offset (eliminate) the carrying amount of the parent's investment in each
subsidiary and the parent's portion of equity of each subsidiary (PFRS 3
Business Combinations explains how to account for any related goodwill) •
eliminate in full intra-group assets and liabilities, equity, income, expenses
and cash flows relating to transactions between entities of the group (profits
or losses resulting from intra-group transactions that are recognized in assets,
such as inventory and fixed assets, are eliminated in full).
A reporting entity includes the income and expenses of a subsidiary in the
consolidated financial statements from the date it gains control until the date when
the reporting entity ceases to control the subsidiary. Income and expenses
of the subsidiary are based on the amounts of the assets and liabilities recognized in the
consolidated financial statements at the acquisition date. The parent and subsidiaries
are required to have the same reporting dates, or consolidation based
on additional financial information prepared by subsidiary, unless impracticable. Where
impracticable, the most recent financial statements of the subsidiary are used, adjusted
for the effects of significant transactions or events between the
reporting dates of the subsidiary and consolidated financial statements. The
difference between the date of the subsidiary's financial statements and that of the
consolidated financial statements shall be no more than three months

5. Non-controlling interests (NCIs). A parent presents non-controlling interests in its


consolidated statement of financial position within equity, separately from the equity
of the owners of the parent.
A reporting entity attributes the profit or loss and each component of other
comprehensive income to the owners of the parent and to the non-controlling
interests. The proportion allocated to the parent and non-controlling interests are
determined on the basis of present ownership interests.

The reporting entity also attributes total comprehensive income to the owners of the
parent and to the non-controlling interests even if this results in the non-controlling interests
having a deficit balance.

6. Preparation of separate financial statements/ Requirement for separate financial


statements

PAS 27 does not mandate which entities produce separate financial statements
available for public use. It applies when an entity prepares separate financial
statements that comply with International Financial Reporting Standards.

Financial statements in which the equity method is applied are not separate
financial statements. Similarly, the financial statements of an entity that does not have a
subsidiary, associate or joint venturer's interest in a joint venture are not separate
financial statements.

An investment entity that is required, throughout the current period and all
comparative periods presented, to apply the exception to consolidation for all of its
subsidiaries in accordance with of PFRS 10 Consolidated Financial Statements presents
separate financial statements as its only financial statements.

[Note: The investment entity consolidation exemption was introduced into PFRS 10 by
Investment Entities, issued on 31 October 2018 and effective for annual periods beginning
on or after 1 January 2020.]

7. Choice of accounting method: Separate Financial Statements of the Parent or


Investor in an Associate or Joint Venture

When an entity prepares separate financial statements, investments in subsidiaries,


associates, and jointly controlled entities are accounted for either:
• at cost, or
• in accordance with PAS 39 [PFRS 9 (effective 2015) Financial
Instruments.!
Such investments may no t be accounted for by the equity method in the parent's/ investor's
separate statements.

The entity applies the same accounting for each category of investments. Investments
that are accounted for at cost and classified as held for sale in accordance with PFRS 5
Non-current Assets Held for Sole and Discontinued Operations are accounted for in
accordance with that PFRS. Investments carried at cost should be measured at the lower of
their carrying amount and fair value less costs to sell. The measurement of investments
accounted for in accordance with PFRS 9 is not changed in such circumstances.

If an entity elects, in accordance with PAS 28 (as amended in 2017), to measure its investments in
associates or joint ventures at fair value through profit or loss in accordance with IFRS 9, it
shall also account for those investments in the same way in its separate financial statements.

Prior to May 2008, both IAS (PAS) No. 27 (Separate and Consolidated Financial Statements) and
IAS (PAS) 18 (Revenue) raised very significant issues of interpretation:
the meaning of 'cost' and
.• the meaning of 'profits...arising from the date of acquisition' (sometimes
referred to as dividends out of pre-acquisition profits which is to be regarded as a recovery of
the investment and therefore accounted for as a reduction of the cost of investment) in the
context of treatment ot dividend income.

As a consequence, in May 2008, the IASB issued amendments to IAS (PAS) 27 relating to the cost of
investment in a subsidiary, jointly controlled entity or associate. These amendments:
• deleted the definition of the cost method from IAS (PAS) 27
• inserted paragraph 38A into IAS (PAS) 27, Paragraph 38A states that:
"An entity shall recognize a dividend from a subsidiary, jointly controlled entity, or associate in
profit or loss in its separate financial statements when its right to receive the dividends is established".

The effect of these changes is that all dividends paid or payable by a subsidiary to a parent are to
be recognized as revenue by the parent. As noted in paragraph BC66H of the Basis of
Conclusions to the amendments, 'the requirement to separate the retained earnings of an entity
into pre-acquisition and posts-acquisition components as a method for assessing whether a
dividend is a recovery of its associated investment' has been removed from IFRSs' (PFRSs').

IAS (PAS) 27 does not define what is meant by 'cost' except in the specific set of
circumstances of certain types of group reorganization and in first-time transition to IFRS (PF^S). IAS
(PAS) 8 - Accounting Policies, Changes, in Estimates and Errors -
requires that in the absence of a specific guidance on IFRS (PFRS), management should first refer
to the requirements and guidance in IFRS (PFRS) dealing with similar and related issues.

IFRS (PFRS) 3 (Revised 2008), the term 'cost' no longer refers to cost of an acquisition so the relevant
measure will be the consideration transferred discussed in Chapter 8.
Another point of reference might be IAS (PAS) 32 - Financial Instruments: Presentation
- and IAS (PAS) 39. Investments in subsidiaries, associates and joint ventures, while
outside the scope of IAS (PAS) 32 and IAS (PAS) 39, are clearly financial assets (and
therefore financial instruments) as defined in those standards.

Instead of the now deleted definition of cost method, entities are now obliged to
apply a two-stage process. Once recognized, all dividends are taken to income and
the parent must now determine whether or not the investment has been impaired
as a result. This list of indicators of impairment in IAS (PAS) 36 as amended includes the
receipt of a dividend from a subsidiary, jointly controlled entity or associate where
there is evidence that:
1. the dividend exceeds the total comprehensive income of the subsidiary,
jointly controlled entity or associate in the period dividend is declared;
or
2. the carrying amount of the investment in the seaprate financial
statements exceedsthe carrying amounts in the consolidated financial
statements of the investee's net assets, including associated goodwill.

Personal Observation: In accounting for investments in subsidiary in the separate/


individual financial statements of parent, the application of the cost method despite the
definition was already deleted in the IAS (PAS) 27 Revised (2008) is still in effect except
that the 'excess of such profits regarded as a recovery of investment and are
regarded as a reduction of investment' was already removed.

8. Investment entities
[Note: The investment entity consolidation exemption was introduced into PFRS 10 by
Investment Entities, issued on 31 October 2018 and effective for annual periods
beginning on or after 1 January 2020.]

If a parent investment entity is required, in accordance with PFRS 10 to measure its


investment in a subsidiary at fair vaiue through profit or loss in accordance with
PFRS 9(2015) or PAS 39 it is required to also account for its investment in a subsidiary in the
same way in its separate financial statements.

When a parent ceases to be an investment entity, the entity can account for an
investment in a subsidiary at cost (based on fair value at the date of change or
status) or in accordance with PFRS 9. When an entity becomes an investment
entity, it accounts for an investment in a subsidiary at fair value through profit or loss
in accordance with PAS 39 or PFRS 9 (2015).

9. Group ReorganizationsSpecified accounting applies in separate financial


statements when a parent reorganizes the structure of its group by establishing a new
entity as its parent in a manner satisfying the following criteria:

• the new parent obtains control of the original parent by issuing equity
instruments in exchange for existing equity instruments of the original parent •
the assets and liabilities of the new group and the original group are the
same immediately before and after the reorganization, and
• the owners of the original parent before the reorganization have the same
absolute and relative interests in the net assets of the original group and the
new group immediately before and after the reorganization.
10. Goodwill or a Gain from Bargain Purchase. For stock acquisition (or acquisition of
shares) in contrast to statutory merger and statutory consolidation (acquisition of
assets and assumption of liabilities) discuss in Chapter 8, the comparison should be
between the following:
i. The sum of:
> the fair value of the consideration transferred
> the recognized amount of any non-controlling interest in
the acquiree
> for a business combination achieved in stages (step
acquisition), the fair value of any previously held equity
interest in the acquiree; and

II. The acquisition-date recognized fair value amount of the identifiable


assets acquired and liabilities assumed.

Goodwill arises when I exceeds II, under:


• Option 1: "Full" Goodwill Method - there is a non-controlling interest
share in the goodwill.
• Option 2: "Partial" Goodwill Method - there is no non-controlling interest
share in the goodwill.

Bargain purchase arises when // exceeds I. When a bargain purchase occurs, a gain
on acquisition is recognized in the profit or loss. While this is consistent with the
pronouncement of PFRS 3 (old) under Option 2, the amount recognized may differ ,
due to the other changes in the PFRS 3 Revised (new) which may also allows
Option 1. It is under Option (1) where there is an inconsistency of recognition of
gar'n, wherein any excess that remains is recognized as a gain, which is attributable only
to the acquirer (or parent company).

11. Acquisitions and disposals that do not result in a change of control.


• Changes in a parent's ownership interest in a subsidiary that do not result in
a loss of control are accounted for as an equity transaction.
> Goodwill is not remeasured
> No gain or loss is recognized on such transactions
• Any difference between the change in the Non-controlling Interests (NCI)
and the fair value of the consideration paid or received is recognized directly
in equity (share premium/APIC) and attributed to the owners of the parent.
12. loss of Control. A parent can lose control of a subsidiary through:
• a sale, or
• distribution, or
• through some other transaction or event in which it takes no part (e.g.
expropriation or the subsidiary being placed in administration or bankruptcy).
• When control is lost
> A gain or loss is recognized in profit or loss
13. Loss of significant influence or joint control.
• Investor loses significant influence over an associate, a gain or loss is
recognized.
• Investor loses joint control over a jointly controlled entity, a gain or loss
is also recognized.
14. Step acquisitions. Business combination leading to acquisition accounting applies
only at the point where control is achieved. This has a number of implications:
• Where the acquirer has a pre-existing equity interest in the entity
acquired. If the acquirer increases its equity interest sufficiently to
achieve control as a "business combination achieved in stages", it must
remeasure its previously-held equity interest in the acquiree at acquisition-
date fair value and recognize the resulting gain or loss, if any, in profit or
loss.
• Once control is achieved: all other increases and decreases in
ownership interests are treated as transactions among equity (refer to Nos. 9,
10 and 11 above) holders and reported within equity. Goodwill does not arise
on any increase; and no gain or loss is recognized on any decrease.
• non-controlling interests are measured on the date of acquisition under
one of the two options (refer to No. 8 above) permitted by PFRS 3
(2008) [par. 19]

15. Reverse Acquisition. A reverse acquisition occurs when the entity that issues
securities (the legal acquirer) is identified as the acquiree for accounting purposes. The
entity whose equity interests are acquired (the legal acquiree) must be the acquirer
for accounting purposes for the transaction to be considered a reverse acquisition. In
this situation, the accounting acquire must meet the definition of a business for the
transaction.

(ff. Accounting for Consolidated Financial Statements (Acquisition Method)

A. Date of Acquisition. To prepare Consolidated Financial Statements, the Investment in


Subsidiary should be eliminated in the consolidated statements.

B. Subsequent to Date of Acquisition

In subsequent to date of acquisition, the newly affiliated companies continue to


maintain their separate accounting records. Furthermore, the eliminations and
adjustments made as part of the consolidation procedures are not entered into the
books of any of the companies; these adjustments are simply "worksheet entries" which
are never formally journalized. As a result, consolidation procedures must be performed
every period in which financial statements are presented. Generally, the
parent company carries its interest in a subsidiary in a single account, "Investment in
Subsidiary." This account is generally carried under one of two methods: the cost
method or financial assets under PAS No. 39. Consolidation procedures are partly
determined by the consolidation method used; however, the final result, that is, the
consolidated statements themselves, must be the same regardless of how the
investment is carried on the parent's books. I urge you now to review the cost .
method of accounting or financial assets for Investments in common stock, in Financial
Accounting / Intermediate Accounting.

C. Transactions Between Affiliated Companies

a. Intercompany Sales of Inventory - Intercompany sales of merchandise create three


problems:
1. The sale and CGS are recorded twice: first, the seller records a sale and
related CGS as the merchandise is "sold" to the affiliatea buyer; secondly, the
buyer resells the goods to outsiders, also recording a sale and CGS. For
consolidated purposes, however, it is obvious that only one sale has occurred.

2. When one company sells merchandise to its affiliote at a price above cost,
the ending inventory of the buyer contains an element of unrealized gross
profit. The gross profit is not realized to the economic entity until it is sold to
outsiders. The preparation of consolidated financial statements requires that
unrealized gross profit be eliminated.

3. Non-controlling interest in the subsidiary's must be based on the sales and


CGS originally reported by the subsidiary. As it was the case in interaffiliate
interest income and- expense, the non-controlling income should reflect the
expense incurred (or revenues obtained) in intercompany transactions. The sale,
however, may not be recognized until after the goods have been sold to an outside
buyer.

b. Intercompany Sales of Fixed Assets - Sales of fixed assets between members of an


affiliated group may result in the recognition of gain or loss by the seller, if the selling price
differs from the carrying amount of the asset. Again, no gain or loss has taken place for
the consolidated entity; assets have merely been transferred from one set of books to
another. Additional complications result from the fact that the buyer of the asset will
record it in its books at the agreed upon purchase price; subsequent depreciation
charges will be based upon this purchase price, thus requiring adjustment. In
summary, an interaffiliate sale of fixed assets involves the following:

1. In the year of sale, restore the carrying amount of the asset to its original BV
and eliminate the gain (loss) recorded by the seller.

2. For each period, adjust depreciation expense and accumulated depreciation


to reflect the original BV of the asset.

3. For periods subsequent to the year of sale. Investment in Subsidiary must be


adjusted to eliminate the gain (loss) contained therein.

a. If the parent is the seller. Investment in Subsidiary absorbs the entire


adjustment.
b. If a less than 100% owned subsidiary is the seller, the adjustment should
be allocated to the Investment in Subsidiary and Non-controlling Interest
- Retained Earnings, Subsidiary, beginning of the year on the basis of their
ownership ratio.

* In some aspect Parent Company's Retained Earnings may be used


depending in how the Parent Company record the transaction in its
books.
c. Receivables and Payable. Intercompany Loans.

1. Receivables and payables - Originate from intercompany transactions such


as the sale of inventory and fixed assets or the rendering of services. These
receivables and payables appear in the affiliated company's trial balance af the
end of the period; note, however, thdt no osset or liability exists outside the
consolidated group. Elimination of the receivable/payable simply involves a
"worksheet entry" reversing the original recording.
2. Intercompany loans - These must also be eliminated from consolidated
statements, in a manner similar to that used for receivables and payables,
above. In addition, interest income and expense and interest accruals must
be eliminated.

d. Combined Financial Statements

1. There are circumstances where combined financial statemenfs (as


distinguished from consolidated statements) of commonly controlled
companies are likely to be more meaningful than their separate
statements. Combined financial statements are often prepared for a group of
related companies (e.g., a group of unconsolidated subsidiaries) or a group of
commonly controlled companies (e.g., one individual owns a controlling
interest in several corporations which are related in their operations).
Consolidated statements are not appropriate if there is no investment by one
affiliate in another to eliminate.

2. Combined financial statements are prepared by combining the individual


companies' financial statement classifications into one set of financial
statements. Intercompany transactions, balances, and profits or losses are
eliminated in the same manner as in consolidated statements. If there are
problems in connection with such matters as minority interests, foreign operations,
different fiscal periods, or income taxes, they are treated in the same manner as
in consolidated statements.
MULTIPLE CHOICE QUESTIONS

Note to the Examinees:

The following ferms should be noted in the worksheet which will lead us to distinguish figures
from:
• Parent's Separate (Internal) Financial statements - the financial statements of parent
before adjustments and working paper elimination entries. It is here wherein the
cosf (initial value) method is used.
• Group/Consolidated Financial Statements - summation of the financial statements
of the group members and the consolidated adjustments. It is wherein adjustments
and eliminating entries are reflected.
• Parent's (Interest/Equity Interest /Non-Controlling Interest) Financial Statements -
the parent figures are then determined by subtracting the Non-controlling Interest
from the total consolidated equity (group/consolidated financial statements).

1. (Adapted: BDO Kendalls) Fair Value of Non-controlling Interest in the


Acquiree (Subsidiary) is not given. Company Z acquires 80% of Company Y
for PI0,000,000, carrying value of Company Y net assets at time of
acquisition being P6,000,000 and fair value of these net identifiable assets
being P8,000,000.

Goodwill arising on consolidation is to be valued on the proportionate


basis or "Partial" Goodwill:

a. PI,600,000 c. P3,600,000
b. P2,000,000 d. P4,500,000

2. Using the same information in No. 1, the amount of non-controlling interest


.arising on consolidation is to be valued on the proportionate basis or
"Partial" Goodwill:

a. PI,200,000 c. P2,500,000
b. PI,600,000 d. P3,000,000

3. Using the same information in No. 1, the amount of goodwill arising on


consolidation is to be valued on the full (fair value) basis or "Full/Gross-
up" Goodwill:

a. PI,600,000 c. P3,600,000
b. P2,000,000 d. P4,500,000
4. Using the same information in No. 1, the amount of non-controlling interest
arising on consolidation is to be valued on the full (fair value) basis or
"Full/Gross-up" Goodwill:

a. PI,200,000 c. P2,500,000
b. PI,600,000 d. P3,000,000

5. (Adapted: Ernst and Young) Fair Value of Non-controlling Interest in the


Acquiree (Subsidiary) is given. Entity Subsidiary has 40% of its share publicly traded
on an exchange. Entity Parent purchases the 60% non-publicly traded shares in
one transaction, paying P6,300,000. Based on the trading price of the shares of
Entity Subsidiary at the date of gaining control a value of P4,000,000 assigned
to the A0% non-controlling interest (or fair value of non-controlling interest),
indicating that Entity Subsidiary has paid a control premium of P300,000. The
fair value of Entity Subsidiary's identifiable net assets is P7,000,000 and a
carrying value of P5,000,000.

Goodwill arising on consolidation is to be valued on the proportionate basis or


"Partial" Goodwill:

a. PI,200,000 c. P3,300,000
b. P2,100,000 d. P4,120,000

6. Using the same information in No. 5, the amount of non-controlling interest


arising on consolidation is to be valued on the proportionate basis or
"Partial" Goodwill:

a. P2,000,000 c. P4,000,000
b. P2,800,000 d. P4,120,000

7. Using the same information in No. 5, the amount of goodwill arising on


consolidation is to be valued on the full (fair value) basis or "Full/Gross-
up" Goodwill:

a. PI,200,000 c. P3,300,000
b. P2,100,000 d. P4,120,000

8. Using the same information in No. 5, the amount of non-controlling interest


arising on consolidation is to be valued on the full (fair value) basis or
"Full/Gross-up" Goodwill:

a. P2,000,000 c. P4,000,000
b. P2,800,000 ' d. P4,120,000
9. (Adapted: Deloitte and Advanced Financial Accounting by Baker, et al.)
Step- Acquisition: Consideration transferred, fair value of Non-controlling Interest
of the acquiree/subsidiary) and Fair value of any previously held equity interest in
the acquiree/subsidiary (step acquisition) is given. Pares Company acquires 15
percent of Serap Company's common stock for P500,000 cash and carries the
investment using the cost method. A few months later, Pares purchases another
60 percent of Serap Company's stock for P2,160,000. At that date, Serap Company
reports identifiable assets with a book value of P3,900,000 and a fair value of
P5,100,000, and it has liabilities with a book value and fair value of PI ,900,000.
The fair value of the 25% non-controlling interest in Serap Company is P900,000.

Goodwill arising on consolidation is to be valued on the proportionate basis or


"Partial" Goodwill:

a. P 84,000 c. P300,000
b. PI 00,000 d. P400,000

10. Using the same information in No. 9, the amount of non-controlling interest
arising on consolidation is to be valued on the proportionate basis or
"Partial" Goodwill:

a. P300,000 c. P800,000
b. P500,000 d. P900,000

11. Using the same information in No. 9, the amount of goodwill arising on
consolidation is to be valued on the full (fair value) basis or "Full/Gross-up"
• Goodwill:

a. P 84,000 c. P300.000
b. PI 00,000 d. P400,000

12. Using the same information in No. 9, the amount of non-controlling interest
arising on consolidation is to be valued on the full (fair value) basis or
"Full/Gross-up" Goodwill:

a. P300,000 c. P800,000
b. P5004000 d. P900,000

13. Using the same information in No. 9, the amount of gain or loss should be
recognized when the additional shares are acquired:

a. Zero c. P40,000loss
b. P40,000gain d. P'68,000 loss
14. (Adapted: Deloitte) Fair Value of Subsidiary is given. Since Fair value of
Subsidiary is given, it already includes all items such as consideration
transferred, fair value of non-controlling interest and any previously held
equity interest in the acquiree. On September 1,2017, Company P acquires 75%
(750,000 ordinary shares) of Company S for P7,500,000 (P10 per share). In the
period around the acquisition date, Company S's shares are trading at about
P8 per share. Company P pays a premium over market because of the
synergies it believes it will get. It its therefore reasonable to conclude that the
fair value of Company S's as a whole may not be PI0,000,000. In fact, an
independent valuation shows that the value of Company S is P9,700,000
(fair value of Company S). Assuming that the fair value of the net identifiable
assets is P8,000,000 (carrying value is P6,000,000)

Goodwill arising on consolidation is to be valued on the proportionate


basis or "Partial" Goodwill:

a. P 200,000 c. PI,700,000
b. PI,500,000 d. P2,000,000

Using the same information in No. 14, the amount of non-controlling interest
arising on consolidation is to be valued on the proportionate basis or
"Partial" Goodwill

a. PI,500,000 c. P2,000,000
b. PI,875,000 d. P2,200,000

16. Using the same information in No. 14, the amount of goodwill arising on
consolidation is to be valued on the full (fair value) basis or "Full/Gross-
up" Goodwill:

a. P 200,000 c. PI,700,000
b. PI,500,000 d. P2,000,000

17. Using the same information in No. 14, the amount of non-controlling interest
arising on consolidation is to be valued on the full (fair value) basis or
"FuJI/Gross-up" Goodwill

a. PI,500,000 c. P2,000.000
b. PI,875,000 d. P2,200,000
18. All the issued and outstanding common stock of Manila Company were
bought by Makati Company on October 1, 2017 for P700,000. The assets
and liabilities of Manila Company were:

Cash P 50,000
Accts. receivable (net of P25,000 allowance
for doubtful accounts) 250,000
Inventory 150,000
Property & equipment (net of PI00,000
allowance for depreciation) 300,000
Accounts/Notes Payable 130,000

On Oct. 1, 2017 the fair value of the following assets were as follows:

Accts. receivable (net) P235,000


Inventory 130,000
Property & equipment (net) 400,000

There is an unrecorded warranty liability on prior-product sales estimated


P20,000 discounted cash flow based on estimated future cash flows.

The amount of goodwill as a result of the business combination should be:

a. P -0- c. P 65,000
b. 35,000 d. 100,000
(PhilCPA)

19. Using the same information in No. 18, the amount of goodwill recorded in
the books of Makati Co. as a result in the business combination should be:

a. P 0 c. P 65,000
b. 35,000 d. 100,000
(PhilCPA)

20. On January 1,2017, Gold Rush Company acquires 80 percent ownership in


California Corporation for P200,000. The fair value of the non-controlling
interest at that time is determined to be P50,000. It reports net assets with a
book value of P200,000 and fair value of P230,000. Gold Rush Company
reports net assets with a book value of P600,000 and a fair value of P650,000 at
that time, excluding its investment in California. What will be the amount of
goodwill that would be reported immediately after the combination
undercurrent accounting practice if the option of full-goodwill method is
used?

a. P50,000 c. P30,000
b. P40,000 d. P20,000
21. The Lampara Company acquired a 70% interest in The Oak Company for
PI,960,000 when the fair value of Oak's identifiable assets and liabilities
was P700,000 and elected to measure the non-controlling interest at its
share of the identifiable net assets. Annual impairment reviews of goodwill
have not resulted in any impairment losses being recognized. Oak's current
statement of financial position shows share capital of pl00,000, a
revaluation reserve of P300,000 and retained earnings of PI,400,000.

Under PFRS 3 Business combinations, what figure in respect of goodwill


should now be carried in Lampara's consolidated statement of financial
position?

a. PI,470,000 c. P700,000
b. PI,260,000 d. PI 60,000

22. The Natural Company acquired 80% of The Loco Company for a
consideration transferred of PI00 million. The consideration was estimated
to include a control premium of P24 million. Loco's net assets were P85
million at the acquisition date. Are the following statements true or false,
according to PFRS 3 Business combinations'?

(1) Goodwill should be measured at P32 million if the non-controlling


interest is measured at its share of Local's net assets.
(2) Goodwill should be measured at P34 million if the non-controlling
interest is measured at fair value.

Statement (I) Statement (2) Statement (I) Statement (2)


a. False False c. True False
b. , False True d. True True

23. The Moon Company acquired a 70% interest in The Swan Company for
PI,420,000 when the fair value of Swan's identifiable assets and liabilities
was PI ,200,000. Moon acquired a 65% interest in The Homer Company for
P300,000 when the fair value of Homer's identifiable assets and liabilities
was P640.000. Moon measures non-controlling interests at the relevant share
of the identifiable net assets at the acquisition date. Neither Swan nor
Homer had any contingent liabilities at the acquisition date and the above
fair values were the same as the carrying amounts in their financial
statements. Annual impairment reviews have not resulted in any impairment
losses being recognized.
Under PFRS 3 Business combinations, what figures in respect of goodwill
and of gains on bargain purchases should be included in Moon's
consolidated statement of financial position?

a. Goodwill: P580,000; Gains on the barain purchses: PI 16,000


b. Goodwill: Nil or zero; Gains on the bargain purchases: PI 16,000
c. Goodwill: Nil or zero; Gains on the bargain purchases: Nil or zero
d. Goodwill: P580,000; Gains on the bargain purchases: Nil or zero

24. On October 1, 2018 The Ting Company acquired 100% of The Green
Company when the fair value of Green's net assets was PI 16 million and
their carrying amount was pi 20 million. The consideration transferred
comprised P200 million in cash transferred at the acquisition date, plus
another P60 million in cash to be transferred 11 months after the acquisition
date if a specified profit target was met by Green. At the acquisition date
there was only a low probability of the profit target being met, so the fair
value of the additional consideration liability was P10 million. In the event,
the profit target was met and the P60 million cash was transferred.

a. P80 million c. P 94 million


b. P84 million d. PI44 million

25. 100% of the equity share capital of The RauCompany was acquired by The
Swift Company on June 30,2018. Swift issued 500,000 new PI ordinary shares
which had a fair vaiue of P8 each at the acquisition date, in addition the
acquisition resulted in Swift incurring fees payable to external advisers of
P200,000 and share issue costs of PI80,000.

In accordance with PFRS 3 Business combinations, goodwill at the acquisition


date is measured by subtracting the identifiable assets acquired and the
liabilities assumed from:

a. P4.00 million c. P4.20 million


b. P4.18 million d. P4.39 million

26. Jones Corporation issues 45,000 shares of previously unissued P10 par value
common stock with a fair market value of P32 per share for net assets of
Dunn Corporation. Jones pays the following costs and expenses related to
the business combination:

Registering and issuing securities PI5,000


Accountants' and legal fees 8,000
Salaries of Jones's employees assigned to the implementation
of the merger 16,000
Cost of closing duplicate facilities 12,000
Cost of shareholders' meeting to vote on the merger.. 5,000
The expenses amounted to:

a. P21,000 c. P41,000
b. 33,000 d. 56,000
(Adapted)

27. Parent Corporation issued 100,000 shares of P20 for common stock for all
the outstanding stock of Subsidiary Corporation in a business combination
consummated as July 1, 2017. Parent Corporation common stock was
selling at P30 per share at the time of the business combination was
consummated. Out-of-pocket costs of the business combination were as
follows:
Finder's fee P50,000
Accountant's fee (advisory) 10,000
Legal fees (advisory) 20,000
Printing costs 5,000
SEC registration costs and fees 12,000
P97,000

The fair value of the consideration transferred accounting will be:

a. P3,097,000 c. P3,017,000
b. 3,080,000 d. 3,000,000
(Adapted)

28. Harrison, Inc. acquires 100% of the voting stock of Rhine Company on
January 1,2017 for P400,000 cash. A contingent payment of P16,500 will be
paid on April 15, 2018 if Rhine generates cash flows from operations of
P27,000 or more in the next year, Harrison estimates that there is a 20%
probability that Rhine will generate at least P27,000 next year and uses an
interest rate of 5% to incorporate the time value of money. The fair value
of PI6,500 at 5%, using a probability weighted approach is P3,142.

What will Harrison record as the acquisition price on January 1, 2017?

a. P400,000 c. P409J42
b. P403,142 d. P416,500
29. Using the same information in No.24, assuming Rhine generates cash flow
from operations of P27,200 in 2017, how will Harrison record the PI 6,500
payment of cash on April 15, 2018?

a. Debit Contingent performance obligation PI6,500 and Credit


Cash PI6,500.
b. Debit Contingent performance obligation P3,142, debit Loss from
contingent performance obligation PI3,358 and Credit Cash
PI 6,500
c. Debit Investment in Subsidiary and Credit Cash, PI 6,500.
d. Debit Goodwill and Credit Cash, PI6,500.

30. Dosmann, Inc. bought all outstanding shares of Lizzi Corporation on January
1, 2017, for P700,000 in cash. The portion of the consideration transferred
results in a fair-value allocation of P35,000 to equipment and goodwill of
P88,000. At the acquisition date, Dosmann also agrees to pay Lizzi's previous
owners and additional PI 10,000 on January 1,2013, if Lizzi earns a 10 percent
return on the fair value of its assets in 2017 and 2018. Lizzi's profits exceed this
threshold in both years. Under which of the following is true?

a. The additional PI 10,000 payment is a reduction in retained


earnings.
b. The fair value of the expected contingent payment increases
goodwill at the acquisition date.
c. Goodwill as of January 1, 2013, increases by P110,000.
d. The PI 10,000 is recorded as an expense in 2013.

31. Lauren Corporation acquired Sarah, Inc. on January 1, 2017, by issuing


13,000 shares of common stock with a P10 per share par value and a P23
market value. This transaction resulted in recording P62,000 of goodwill.
Lauren also agreed to compensate Sarah's former owners for any difference if
Lauren's stock is worth less than P23 on January 1, 1012. On January 1, 2018,
Lauren issues an additional 3,000 shares to Sarah's former owners to honor the
contingent consideration agreement. Under which of the following is
true?

a. The fair value of the expected number of shares to be issued for


the contingency increases the Goodwill account balance at .
the date of acquisition.
b. The Investment account balance is not affected, but the parent's
Additional Paid-in Capital is reduced by the par value of the
extra 3,000 shares when issued.
c. All of the subsidiary's assets and liability accounts must be revalued
for consolidation purposes based on their fair values as of January
1,2013.
d. The additional shares are assumed to have been issued on January
1, 2017, so that a retrospective adjustment is required.
32. (Adapted: Advanced Financial Accounting by Baker, et al.) Parlor
Company acquires 75 percent of Saloon Company's common stock for
P225,000 cash. At that date, the non-controlling interest in Saloon has a book
value of P52,500 and a fair value of P82,000. Also on that date, Saloon
reports identifiable assets with a book value of P400,000 and a fair value of
P510,000, and it has liabilities with a book value and fair value of PI 90,000.

Gain on bargain purchase arising on consolidation if fair value of net


identifiable assets is to be valued on the proportionate basis:

a. Zero c. P 1-5,000
b. P13,000 d. PI 7,333

33. Using the same information in No. 32, compute the gain on bargain
purchase arising on consolidation if fair value of net identifiable assets is
to be valued on the full (fair value) basis.

a. Zero c. PI 5,000
b. PI 3,000 d. PI 7,333

34. (Step-Acquisition). Seminarian, Inc. has 100,000 shares of P2 par value stock
outstanding. Priests Corporation acquired 30,000 shares of Seminarian's shares
on January 1,2017 for PI 20,000 when Seminarian's net assets had a total fair
value of P350,000. On July 1, 2020, Priests agreed to buy an
additional 60,000 shares of Seminarian from single stockholder for P6 per share.
Although Seminarian's share s were selling in the P5'range around July 1, 2020,
Priests forecasted that obtaining control of Seminarian would produce significant
revenue synergies to justify the premium price paid. If Seminarian's net
identifiable assets had a fair value of P500,000 on July 1, 2020, how much
goodwill on full fair value basis should Priests report in its post-combination
consolidated balance sheet?

a. P 0 c. P 90,000
b. P60,000 d. PI 00,000

Sale of Subsidiary - Loss of Control of a Subsidiary or "Deconsolidation"

35. (Adapted: Ernst and Young) Entity P has a 90% controlling interest in Entity
S. On December31,2017, the carrying value of Entity S's net assets in Entity P's
consolidated financial statements is P100,000 and the carrying amount
attributable to the non-controlling interest's in Entity S (including the non-
controlling interest's share of accumulated other comprehensive income)
is PI0,000. On January 1, 2018, Entity P sells 80% of the share in Entity S to a third
party for cash proceeds of PI20,000. As a result of the sale. Entity P loses
control of Entity S but retains a 10% non-cc ntrolling interest in Entity S. The fair
value of the retained interest on that date is PI2,000.

Determine the gain or loss on disposal (or deconsolidation) should be:

a. P20,000gain c. P42,000gain
b. P32,000gain d. P42,000 loss

36. (Adapted: Advanced Financial Accounting by Baker, et al.) Pedro


Company owns 80,000 shares of Santa Corporation's 100,000 outstanding
common shares, acquired at book value. The December 31, 2017,
consolidated balance sheet presented by Pedro and Santa included net
assets of Santa in the amount of P600,000. On January 1, 2018, Pedro sells
70,000 shares of Santa for P490,000. The fair value of Pedro's remaining 10%
interest in Santa is P70.000. What amount of gain or loss, if any, should be
recognized on the sale of Pedro's shares resulting in deconsolidation, and
how much of that should be attributed to Pedro?

Determine the gain or loss on disposal (or deconsolidation) should be:

a. P40,000loss c. PI 0,000 gain


b. P80,000loss d. P80,000gain

37. Parent Corporation owns an 85% interest in Subsidiary Corporation. On


December 31,2017 in the Parent's consolidated financial statements the
carrying value of Subsidiary's net assets is PI,000,000 and the carrying
value of the non-controlling interest in Subsidiary (including the non-
controlling interest's share of accumulatea other comprehensive income) is
PI00,000. On January 1,2018, Parent Corporation decided to sell a 50%
interest in Subsidiary to a third party in exchange for cash of P600,000. As a
result of this transaction, Parent loses control of Subsidiary but retains a
35% interest in the former subsidiary, valued at P350,000 on that date.

Determine the gain or loss on disposal (or deconsolidation) should be:

a. P50,000loss c. P300,000gain
b. P50,000gain d. P300,000loss
Sale of Subsidiary - Not Resulting in Loss of Control

38. (Adapted: BDO Kendalls) No Additional Shares Issued. Parentis Ltd. has
an 80% investment in Salentis Ltd. with a carrying amount of P80,000,000. The
fair value of Salentis Ltd. is P200,000,000. The following year, Parentis Ltd.
decided to sell a 29% interest in Subsidiary to a third party in exchange for cash.

Determine the gain or loss on disposal of shares to be recognize in the profit


or loss statement:

a. Zero c. P29,000,000 loss


b. P29,000,000 gain d. P 3,000,000 loss

39. (Adapted: Advanced Financial Accounting by Baker, et al.) No Additional


Shares Issued. Padyak Company owns 80,000 shares of Sirkulo Corporation's
100,000 outstanding common shares, acquired at book value. The
December 31,2017, consolidated balance sheet presented by Padyak and Sirkulo
included net assets of Sirkulo in the amount of P600,000. On January 1, 2018,
Padyak sells 10,000 shares (10%) of its Sirkulo stock to unrelated
parties for P70,000.

Determine the gain or loss on disposal of shares to be recognized in the profit


or loss statement:

a. Zero c. PIO.OOO loss


b. P10,000gain d. P 5,000 loss

40. (Adapted: Advanced Financial Accounting by Baker, et al.) With


Additional Shares Issued. Padyak Company owns 80,000 shares of Sirkulo
Corporation's 100,000 outstanding common shares, acquired at book value.
The December 31,2017, consolidated balance sheet presented by Padyak and
Sirkulo included net assets of Sirkulo in the amount of P600,000. On January
1,2018, Sirkulo issues 25,000 additional shares of common stock to unrelated
parties for PI 75,000.
*
The amount to be credited to "additional paid-in capital/share premium"
account:

a. Zero c. P 55,000
b. P 16,000 d. P104,000
41. Baning, Inc. buys 60% of the outstanding stock of Gra, Inc. in an acquisition
that resulted in the acquisition of goodwill. Gra owns a piece of land that
cost P200,000 but was worth P500.000 at the acquisition date. What value
should be attributed to this land in a consolidated balance sheet at the
date of takeover?

a. PI 20,000 c. P380,000
b. P300,000 d. P500,000

42. Paro Company purchased 80% of the voting common stock of Sabon
Company for P900,000. There are no liabilities. The following book and
fair values are available for Sabon:

Book Value Fair Value


Current assets P100,000 P200,000
Land and building 200,000 200,000
Machinery 300,000 600,000
Goodwill 100,000 ?

The machinery will appear on the consolidated balance sheet at

a. P600,000 c. P480,000
b. P540,000 d. P300,000

43. Pagach Company purchased 80% of the voting common stock of Rage
Company for P1,800,000. The following book and fair values are available:

Book Value Fair Value


Current assets P 150,000 P 300,000
Land and building 280,000 280,000
Machinery 400,000 700,000
Bonds payable (300,000) (250,000)
Goodwill 150,000 ?

The bonds payable will appear on the consolidated balance sheet:

a. at P300,000 (with no premium or discount shown).


b. at P300,000 less a discount of $50,000.
c. at P0; assets are recorded net of liabilities.
d. at P300,000 less a discount of P40,000.
44. Chapel Hill Company had common stock of P350,000 and retained earnings
of P490,000. Blue Town Inc. had common stock of P700,000 and retained earnings
of P980,000. On January 1,2017, Blue Town issued 34,000 shares of common stock
with a P12 par value and a P35 fair value for all of Chapel Hill Company's
outstanding common stock. This combination was accounted for as an
acquisition. Immediately after the combination, what was the consolidated net
asset?

a. P2,870,000 c. PI,680,000
b. P2,520,000 d. PI, 190,000

45. Beta Company acquired 100 percent of the voting common shares of
Standard Video Corporation, its bitter rival, by issuing bonds with a par value
and fair value of PI50,000. Immediately prior to the acquisition, Beta
reported total assets of P500,000, liabilities of P280,000, and stockholders'
equity of P220,000. At that date. Standard Video reported total assets of
P400,000, liabilities of P250,000, and stockholders' equity of P150,000. Included in
Standard's liabilities was an account payable to Beta in the amount of P20,000,
which Beta included in its accounts receivable.

Based on the preceding information, what amount of total assets did Beta
report in its balance sheet immediately after the acquisition?

a. P500,000 c. P750,000
b. P650,000 d. P900,000

46. Using the same information in No. 45, what amount of total assets was
reported in the consolidated balance sheet immediately after acquisition?

a. P650,000 c. P920,000
b. P880,000 d. P750,000
47. The financial statements for Goodwin, Inc. and Corr Company for the
year ended December 31,2017, prior to Goodwin's business combination
transaction regarding Corr, follow (in thousands):

Goodwin Corr
Revenues ::... P 2,700 P 600
Expenses 1,980 400
Net Income P 720 P 200

Retained earnings, 1/1 P 2,400 P 400


Net income 720 200
Dividends ( 270) ( 0)
Retained earnings, 12/31 P 2,850 P 600

Cash P 240 P 220


Receivables and inventory 1,200 340
Buildings (net) 2,700 600
Equipment (net) 2,100 1,200
Total assets P 6,240 P 2,360

Liabilities P 1,500 P 820


Common stock 1,080 400
Additional paid-in capital 810 540
Retained earnings 2,850 600
Total liabilities and stockholders' equity..... P 6,240 P 2,360

On December 31, 2017, Goodwin issued P600 in debt and 30 shares of its P10
par value common stock to the owners of Corr to purchase all of the
outstanding shares of that company. Goodwin shares had a fair value of P40
per-share. Goodwin paid P25 to a broker for arranging the transaction. Goodwin
paid P35 in stock issuance costs. Corr's equipment was actually worth PI,400
but its buildings were only valued at P560.

What amount is the investment recorded on Goodwin's books?

a. PI,540 c. PI,825
b. PI,800 d. PI,860
48. Using the same information in No. 47, compute the consolidated
revenues for 2017.

a. P3,300 c. PI,540
b. P2,700 d. P 720

49. Using the same information in No. 47, compute the consolidated expenses
for 2017:

a. PI,980 c. P2,015
b. P2,005 d. P2,040

50. Using the same information in No. 47, compute the consolidated cash
account at December 31,2017.

a. P460 c. P425
b. P435 d. P400

51. Using the same information in No. 47, compute the consolidated buildings
(net) account at December 31,2017:

a. P2,700 c. P3,260
b. P3,370 d. P3,300

52. Using the same information in No. 47, compute the consolidated goodwill
account at December 31,2017:

a. P 0 c. P125
b. P100 d. P160

53. Using the same information in No. 47, compute the consolidated common
stock account at December 31,2017:

a. PI,080 c. PI,480
b. PI,380 d. P2,280

54. USing the same information in No. 47, compute the consolidated additional
paid-in capital at December 31,2017:

a. P 810 c. PI,675
b. Pl,350 d. Pl,910
55. Using the same information in No. 47, compute the consolidated retained
earnings at December 31,2017:

a. P2,800 c. P2,850
b. P2,825 d. P3,425

(Adapted: Applying IFRS - 20Q9 Ed. by Alfredson, et. al.)


56. On July 1,2017, Parent Ltd. acquired all the issued share capital of Sub Ltd.
giving in exchange of 100,000 shares in Parent Ltd. these having a fair value of P5
per share. At acquisition date, the balance sheets of Parent Ltd. and Sub Ltd. and
the fair values of Sub Ltd's assets and liabilities, were as follows: (refer to next page)

Parent Ltd. Sub Ltd.


Carrying Carrying Fair
Amount Amount Value
EQUITY AND LIABILITIES
Equity
Share capital P550,000 P300,000
Retained earnings 350,000 140,000
Total equity P900,000 P440.000
Liabilities
Provisions P 30,000 P 60,000 P 60,000
Payables 27,000 34,000 34,000
Tax liabilities 10,000 6,000 6,000
Total liabilities P 67,000 PI00,000
Total equity and liabilities P967,000 P54O000

ASSETS
Land P120..000 P150,000 PI 70.000
Equipment 620,000 480,000 330,000
Accumulated depreciation. ( 180,000) ( 170,000)
Investment in Subsidiary
(Shares in sub Ltd) 500,000
Inventory 92,000 75,000 80,000
Cash 15,000 5,000 5,000
Total assets P967,000 P540,000
At acquisition date, Sub Ltd. has an unrecorded patent with a fair value of
P20,000 and a contingent liability of with a fair value of PI5,000. The tax rate
is 30%.

The amount of goodwill acquired on July 1, 2017:

a. P25,000 c. PI 0,000
b. 15,000 d. Zero

57. Using the same information in No. 56, consider the same situation where
the assets recorded by the subsidiary at acquisition date are the same as
presented above, except that now there is recorded goodwill, as follows:

Sub Ltd.
Carrying Fair
Amount Vafue
Cash P 5,000 P 5,000
Land 150,000 170,000
Equipment 480,000 330,000
Accumulated depreciation ( 170,000)
Goodwill 10,000
Inventory.. 75,000 80,000
P550,000

Assume that the retained earnings balance is P150,000 rather than P140,000.

The amount of goodwill in the consolidated balance sheet on July 1, 2017:

a. P25,000 c. PI 0,000
b. 15,000 d. Zero

58. Using the same information in Nos. 56 and 57, the amount of unrecorded
goodwill acquired on July 1, 2017:

a. P25,000 c. PI 0,000
b.. 15,000 d. Zero

59. Using the same information in No. 56, and one of the payables at acquisition
date is a dividend payable of P8,000. The parent acquires the shares in the
subsidiary on a cum div. basis or "dividends-on" arrangement. The amount of
goodwill acquired on July 1, 2017:

a. 27,000 c. PI 2,000
b. 17,000 d. Zero
60. On January 1, 2017, Park Corporation and Strana Corporation and their
condensed balance sheet are as follows:

Park Corp. Strand Corp.


Current Assets P 70,000 P 20,000
Non-current Assets 90,000 40,000
Total Assets PI 60,000 P 60,000
Current Liabilities P 30,000 P 10,000
Long-term Debt 50,000
Stockholders' Equity 80,000 50,000
Total Liabilities and Equities PI60,000 P 60,000

On January 2, 2017, Park Corporation borrowed P60,000 and used the


proceeds to obtain 80% of the outstanding common shares of Strand
Corporation. The acquisition price was considered proportionate to Strand's fair
value. The P60,000 debt is payable in 10 equal annual principal payments, plus
interest, beginning December 31, 2017. The excess fair value of the investment
over the underlying book value of the acquired net assets is allocated to
inventory (60%) and to goodwill (40%).

On a consolidated balance sheet as of January 2, 2017, what should be the


amount for each of the following?

The amount of goodwill using proportionate basis (partial):

a. P 0 c. PI 0,000
b. P 8,000 d. P20,000

61. Using the same information in No. 60, the amount of goodwill using full
fair value (full/gross-up) basis:
a. P 0 c. PI 0,000
b. P 8,000 d. P20.000

62. Using the same information in No. 60, the amount of current assets should
be:
a. P105,000 c. P 100.000
b. P102,000 d. P 90,000
63. Using the same information in No. 60, the amount of non-current asset
using proportionate basis (partial) in computing goodwill should be:
a. PI 30,000 c. PI 38,000
b. P134,000 d. P140,000

64. Using the same information in No. 60, the amount of non-current assets
using full fair value basis (full/gross-up) in computing goodwill should be:
a. P130,000 , a P138,000 . '.
b. PI 34,000 d. PI 40,000

65. Using the same information in No. 60, the amount of current liabilities
should be:
a. P 50,000 c. P 40,000
b. P 46,000 d. P 30,000

66. Using the same information in No. 60, the amount of non-current liabilities
should be:

a. PI 10,000 c. P 90,000
b. PI 04,000 d. P 50,000

67. Using the same information in No. 60, the amount of stockholders' equity
using proportionate (partial goodwill) basis to determine non-controlling
interest should be:

a. P 80,000 c. P 95,000
b. P 93,000 d. PI 30,000

68. Using the same information in No. 60, the amount of stockholders' equity
using full fair value (full/gross-up goodwill) proportionate basis to determine
non-controlling interest should be:

a. P 80,000 c. P 95,000
b. P 93,000 d. PI 30,000

69. On January 1,2017, Colorado Corporation acquired 75 percent of Denver


Company's voting common stock for P90,000 cash. At that date, the fair value
of the noncontrolling interest was P30,000. Denvers's balance sheet at the date
of acquisition contained the following balances:

Common Stock P 100,000


Additional paid-in capital 20,000
Retained earnings (deficit) ( 20,000)
At the date of acquisition, the reported book values of Denver's assets and
liabilities approximated fair value. Eliminating entries are being made to
prepare a consolidated balance sheet immediately following the business
combination.
Based on the preceding information, in the entry to eliminate the
investment balance,

a. retained earnings (deficit) will be credited for P20,000.


b. additional paid-in-capital will be credited for P20,000.
c. differential will be credited for PI0,000.
d. noncontrolling interest will be debited for P30,000.

Reverse Acquisition (Adapted: Applying IFRS - 2009 Ed. by Alfredson, et. al.)

70. The balance sheets of Pedro Ltd. and Santi Ltd. on June 30, 2017 were as
follows:

Pedro Ltd Santi Ltd


Current assets P 500 P 700
Non-current assets 1,300 3,000
Total assets PI,800 P3,700

Share capital:
lOOshares P 300
60 shares P 600
Retained earnings 800 1,400
PI,100 P2,000

Current liabilities P 300 P 600


Non-current liabilities 400 1,100
P 700 PI,700 '
Total equity and liabilities PI,800 P3,700

On July 1, 2017, Pedro Ltd acquired all the issued shares of Santi Ltd giving in
exchange 2 1/2 Pedro Ltd shares for each ordinary share of Santi Ltd. Pedro
Ltd thus issued 150 shares to acquire the 60 shares issued by Santi Ltd.

The fair value of each ordinary share of Santi Ltd on July 1, 2017 is P40, while the
quoted market price of Pedro Ltd's ordinary shares is P16. The fair values of Pedro
Ltd's identifiable assets and liabilities at acquisition date are the same as their
carrying amounts except for the non-current assets whose fair value was PI ,
500. The tax rate is 30%.

The amount of goodwill acquired on July 1, 2017:

a. PI,160 c. P400
b. 856 d. 360
Reverse Acquisition
71. Mask, a private limited company, has arranged for Man, a public limited
company, to acquire if as a means of obtaining a stock exchange listing. Man
issues 15 million shares to acquire the whole of the share capital of Mask (6 million
shares). The fair value of the net assets of Mask and Man are P30 million and PI 8 million
respectively. The fair value of each of the shares of Mask is P6 and the quoted market
price of Man's shares is P2. The share capital of Man is 25 million shares after the
acquisition. Calculate the value of goodwill in the above acquisition.

a. PI6 million c. P10 million


b. PI2 million d.. P 6 million

Subsequent to Date of Acquisition


72. Watkins, Inc. acquires all of the outstanding stock of Glen Corporation on
January 1, 2017. At that date, Glen owns only three assets and has no liabilities:

Book Value Fair value


Inventory P 40,000 P 50,000
Equipment 10-year life) 80,000 75,000
Building (20-year life) 200,000 300,000

If Watkins pays P450,000 in cash for Glen, what amount would be


represented as the subsidiary's Building in a consolidation at December
31, 2013, assuming the book value at that date is still P200,000?

a. P200,000 c. P285,000
b. P255,000 d. P300,000

73. Using the same information in No. 72, if Watkins pays P400,000 in cash for
Glen, what amount would be represented as the subsidiary's Building in a
consolidation at December 31, 2013, assuming the book value at that date is still
P200,000?

a. P200,000 c. P285,000
b. P255,O00 d. P300,000

74. Usirlg the same information in No. 72, if Watkins pays P450,000 in cash for
Glen, what amount would be represented as the subsidiary's Equipment in a
consolidation at December 31, 2013, assuming the book value at that date is still
P80,000?

a. P70,000 c. P76,500
b. P73,500 d. P80,000
75. Using the same information in No. 72, if Watkins pays P450,000 in cash for
Glen, what allocation should be assigned to the subsidiary's Equipment in
preparing for consolidation at December 31, 2013, assuming the book
value at that date is still P80,000?

a. P 3,500 c. P75,000
b. P 5,000 d. P80,000

76. On January 1,2017, Brendan, Inc. reports net assets of P760,000 although
(equipment with a four-year life) having a book value of P440,000 is worth
P500,000 and unrecorded patent is valued at P45,000. Brandon Corporation
pays P692,000 on that date for an 80 percent ownership in Brendan. If the
patent is to be written-off over a 10-year period, at what amount should it
be reported on consolidated statements at December 31, 2018?

a. P28,000 c. P36,000
b. P32,400 d. P40,500

77. On January 1, 2017, Turner, Inc. reports net assets of P480,000 although a
building (with a 10-year life) having a book value of P260,000 is now worth
P300,000. Renrut Corporation pays P540,000 on that date for a 90 percent
ownership interest in Turner. On December 31,2013, Turner reports a Building
account of PI82,000 and Renrut reports a Building account of P510,000.
What is the consolidated balance of the Building account?

a. P720,000 c. P780,000
b. P724,000 d. P810,000

78. On January 1, 2017, Harry, Inc. reports net assets of P880,000 although a
patent (with a 10-year life) having a book value of P330,000 is now worth
P400.000. Newt Corporation pays P840,000 on that date for an 80 percent
ownership in Newt. On December 31,2018, Harry reports total expenses of
P621,000 while Newt reports expenses of P714,000. What is the consolidated
total expense balance on December 31,2018?

a. PI, 197-800 c. PI,342,000


b. PI,335,000 d. PI,349,000
79. McGuire company acquired 90 percent of Hogan Company on January 1,
201 l,forP234,000cash. Hogan's stockholders' equity consisted of common
stock of PI 60,000 and retained earnings of P80,000. An analysis of Hogan's net
assets revealed the following:

Any excess consideration transferred over fair value is attributable to an


unamortized patent with a useful life of 5 years.

Book Value Fair value


Building (10-year life) P 10,000 P 8,000
Equipment(4-vearlife) 14,000 18,000
Land 5,000 12,000

In consolidation at January 1, 2017, what adjustment is necessary for


Hogan's Buildings account?

a. P2,000 increase c. PI,800 increase


b. P2,000 decrease d. PI,800 decrease

80. In consolidation at December 31, 2017, what adjustment is necessary for


Hogan's Buildings account?

a. PI,620 increase c. PI,800 increase


b. PI,620 decrease d. PI,800 decrease

81. In consolidation at January 1, 2017, what adjustment is necessary for


Hogan's Land account?

a. P7,000 increase c, P6,300 increase


b. P7,000 decrease d. P6,300 decrease

82. In consolidation at December 31, 2018, what adjustment is necessary for


Hogan's Land account?

a. P 0 c. P6,300 increase
' b. P7,000 increase d. P6,300 decrease
83. Bell Company acquires 80% of Demers Company for P500,000 on January
1,2017. Demers reported common stock of P300,000 and retained earnings of
P200,000 on that date. Equipment was undervalued by P30.000 and
buildings were undervalued by P40,000, each having a 10-year remaining life.
Any excess consideration transferred over fair value was attributed to
goodwill with an indefinite life. Based on an annual review, goodwill has not
been impaired.

Demers earn income and pays dividends as follows:

2017 2018 2013


Net income P 100,000 P120,000 P130.000
Dividends 40,000 50,000 60.000

Assume the initial value method (or cost method) is applied.

Compute Bell's Investment in Demers at December 31, 2017.

a. P500.000 c. P574.400
b. P542.400 d. P625.000

84. Using the same information in No. 83, compute Bell's Investment in Demers
at December 31,2013.

a. P676,O00 c. P592,400
b. P625.000 d. P500.000

85. Using the same information in No. 83, how much does Bell report as Income
from Demers/ Dividend Income for the year ended December 31, 2013?

a. P48,000 c. P56,000
b. P5O400 d. P98.400

86. Using the same information in No. 83, compute the non-controlling
interest in the net income of Demers at December 31, 2018.

a.- PI4,000 c. P22,600


b. PI 8,400 d. P24.000

87. Using the same information in No. 83, compute the non-controlling interest
of Demers using full-goodwill method at December 31, 2013.
a. P 80,000 c. PI40,000
b. PI 07,800 d. PI 60,800

88. On January 1, 2017, Wilt Corporation pays P388.000 for a 60 percent


ownership in Chamberlain. Annual excess fair value amortization of PI 5,000 results
from the acquisition. On December 31, 2018, Chamberlain reports revenues of
P400,000 and expenses of P300,000 and Wilt reports revenues of P700,000 and
expenses of P400,000. The parent figures contain no income from the subsidiary.
What is the consolidated net income attributable to the controlling interest /
profit attributable to equity holders of parent?

a. P231,000 c. P366,000
b. P351,000 d. P400,000

89. On January 1,2017, Parent Company purchased 80% of the common stock
of Subsidiary Company for P316,000. On this date. Subsidiary Company had
common stock, other paid-in capital, and retained earnings of P40,000, PI20,000,
and PI90,000, respectively. Parent Company's common stock amounted to
P500,000 and retained earnings of P200,000.

On January 1, 2017, the only tangible assets of Subsidiary that were


undervalued were inventory and building. Inventory, for which FIFO is used, was
worth P5,000 more than cost. The inventory was sold in 2017. Building, which was
worth PI5,000 more than book value, has a remaining life of 8 years, and
straight-line depreciation is used. Any remaining excess is fullgoodwill with an
impairment for 2017 amounting to P3,000. Subsidiary Company reported net
income of P50,000 and paid dividends of PI0,000 in 2017,while the parent's
reported net income amounted to PI00,000 and paid dividends of P20,000.

Determine the Consolidated Net Income Attributable to Controlling Interest / Profit


Attributable to Equity Holders of Parent:

a. PI 42,000 c. PI 26,500
b. P132.125 d. P124J00

90. Using the same information in No. 89, compute the Consolidated Net
Income Attributable to Controlling Interest / Profit Attributable to Equity
Holders of Parent:

a. PI 42,000 c. PI 26,500
b. P132.125 d. P124J00
91. Using the same information in No. 89, compute the Non-controlling in
Net Income / CNI attributable to Non-controlling interest:

Partial Full Partial Full


Goodwill Goodwill Goodwill Goodwill
a. PI 0,000 PI 0,000 c. P8,025 P8,625
b. R 8,625 P 8,625 -d. P8.625 P8-025

92. Using the same information in No. 89, compute the Equity Holders of Parent
- Retained Earnings / Controlling Interest in the Consolidated Retained
. Earnings:

a. P20O000 c. P324,100
b. P304,100 d. P342.125

93. Using the same information in No. 89, compute the Consolidated/Group
Retained Earnings on full-goodwill approach:

a. P200,000 c. P324,100
b. P304,100 d. P342,125

94. On January 1,2017, Parent Company acquired 90% of Subsidiary Company


in exchange for 5,400 shares of P10 par common stock having a market value of
P120,600. Parent and Subsidiary condensed balance sheets were as follows:

Parent Company and Subsidiary Company


Balance Sheets at January 1,2017
(before combination)
Parent Subsidiary
Company Company
Assets
Cash P 30,900 P 37,400
Accounts receivable (net) 34,200 9,100
Inventories 22,900 16,100
Equipment (net) 179,000 40,000
Patents z 10,000
Total assets P267,000 PI 12,600
Liabilities and stockholders' equity
Accounts payable P4.000 P6,600
Bonds payable, 10% 100,000
Common stock, P10 par 100,000 50,000
Additional paid-in capital 15,000 15,000
Retained earnings 48,000 41,000
Total liabilities and stockholders' equity P267,000 PI 12,600

At the date of acquisition, all assets and liabilities of Subsidiary Company


have book value approximately equal to their respective market values
except the following as determined by appraisal as follows:

Inventories (FIFO method) P17.100


Equipment (net - remaining life - 4 yrs.) 48,000
Patents (remaining life 10 yrs.) 13,000
Goodwill (no impairment)

Compute the amount of partial goodwill on January 1, 2017:

a. P2,600 c. PI 4,400
b. 3,800 d. 25,200

95. Using the same information in No. 94, compute the non-controlling interests
(in net assets) on January 1,2017.

a. PI 0.600 c. PI 1,800
b. 11,200 d. 13,090

96. Using the same information in No. 94, compute the Consolidated Retained
Earnings, January 1,2017:

a. P48,000 c. P84,900
b. 52,100 d. 89,000

97. Using the same information in No. 94, compute the Equity Holders of Parent
• - Retained Earnings, January 1,2017:

a. P48,000 c. P84,9O0
b. 52,100 d. 89,000
98. In addition to the information in No. 94, assuming that on December 31,
2017, the following results were given:
Dividends Net
Paid Income
Parent Company : P15,000 P30,200
Subsidiary Company 4,000 9,400

Using cost method to record results of operations, compute the


investment balance on December 31, 2017:

a. P -0- c. PI 22,160
b. 120,600 d. 125,460

99. Using the same information in Nos. 94 and 98, compute Dividend Income
for 2017 using cost method:

a. P -0- c. P4,000
b. 3,600 d. 8,400

100. Using the same information in Nos. 94 and 98, compute the Non-controlling
Interest in Net Income on December 31, 2017:

a. P-0- c. P610
b. 540 d. 940

101. Using the same information in Nos. 94 and 98, compute the Non-controlling
Interests on December 31,2017:

a. P10,600 c. P12,010
b. 11,140 d. 12,300

102. Using the same information in Nos. 94 and 98, compute the Profit for the
period attributable to Equity Holders of Parent on December 31, 2017:

a. P26,600 c. P36,000
b. 32,090 d. 44,100
*
103. Using the same information in Nos. 94 and 98, compute the Consolidated/
Group Net Income on December 31, 2017:

a. P26,600 c. P32,700
b. 32,090 d. 44,100
104. Using the same information in Nos. 94 and 98, compute the Consolidated
Retained Earning, December 31, 2017:
a. P64,760 c. P69,400
b. 65,090 d. 69,800

105. Using the same information in Nos. 94 and 98, compute the Equity Holders
of Parent - Retained Earnings, December 31, 2017.
a. P64,760 c. P69,400
b. 65,090 d. 69,800

106. Using the same information in Nos. 94 and 98. compute the Consolidated
Total Equity (Stockholders' Equity) on December 31, 2017.
a. PI 08,090 c. P312,700
b. 300,690 d. 317,410

107. (Push-down Accounting vs. No push-down). Prince Company acquires


Duchess, Inc. on January 1, 2017. The consideration transferred exceeds
the fair value of Duchess' net assets. On that date, Prince has a building
with a book value of PI,200,000 and a fair value of PI,500,000. Duchess
has a building with a book value of P400,000 and fair value of P500,000.

What amounts in the Building account appear on Duchess' separate


balance sheet and on the consolidated balance sheet immediately
after acquisition?
No push-down Accounting
Push-down Accounting
P500,000 and P2,000,000
a. P400,000 and P1,600,000 P400,000 and P1,700,000
b. P500,000 and P1,700,000 P500,000 and P1,700,000
c. P400,000 and P1,700,000 P400,000 and P2,000,000
d. P500,000 and P2,000,000
109. Using the same information in No. 108 compute the Consolidated Net
Income (CNI) and the Non-controlling Interests in Net Income (NCINI)

CNI NCINI CNI NCINI


a. P394,0O0 PI 8,800 c. P373.300 P 18,800
b. P372,850 P21.150 d. P394,000 P 21,150

110. On April 1, 2017, Nokia, Inc. exchanges P430,000 fair-value consideration


for 70 percent of the outstanding stock of Ericsson Corporation. The
remaining 30 percent of the outstanding shares continued to trade at a
collective fair value of PI 65,000. Ericsson's identifiable assets and liabilities
each had book values that equaled their fair values on April 1 for a net 'total
of P500,000. During the remainder of the year, Ericsson generates revenues of
P600,000 and expenses of P360,000 and paid no dividends. On
a December 31 consolidated balance sheet, what amount should be
reported as non-controlling interest on a full-fair value basis?

a. P219,000 c. P237,000
b. P237,0O0 d. P250,500

111. On January 1,2017, Payne Corp. purchased 70% of Shayne Corp.'s P10 par
common stock for P900,000. On this date, the carrying amount of Shayne's net
assets was PI,000,000. The fair values of Shayne's identifiable assets and
liabilities were the same as their carrying amounts except for plant assets
(net), which were P200,000 in excess of the carrying amount. For the year ended
December 31,2017, Shayne had net income of PI 50,000 and paid gash
dividends totaling P90,000. Excess attributable to plant assets is amortized over
10 years.

In the December 31, 2017, consolidated balance sheet, non-controlling


interest on a full-fair value basis should be reported at

a. P282,714 c. P345,500
b. P300,500 d. P397,714
116. The White Company acquired an 80% interest in The Pulley Company when
Pulley's equity comprised share capital of PI00,000 and retained earnings of
P500,000. Pulley's current statement of financial position shows share capital of
P100,000, a revaluation reserve of P400,000 and retained earnings of PI,400,000.
What figure in respect of Pulley's retained earnings should be included in the
consolidated statement of financial position?

a. P 720,000 c. PI,040,000
b. PI,440,000 d. PI,520,000

117. On January 2, 2017, Par Company purchased 75% of Sub Company's


outstanding common stock. Selected balance sheet data at December
31, 2017 is as follows:

• Par Sub
Total assets P420,000 P180,000

Liabilities PI20,000 P 60,000


Common stock 100,000 50,000
Retained earnings 200,000 70,000
P420,000 PI 80,000

During 2017, Par and Sub paid cash dividends of P25,000 and P5,000,
respectively, to their shareholders. There were no other intercompany
transactions.

On December 31, 2017, in the consolidated/group financial statements, what


amount should Par report as consolidated:

Dividends Non-controlling interest


paid in net assets
a. P30.000 PI 20,000
b. 30,000 30,000
c. 25,000 30,000
d. 26,250 30,000

18. Using the same information in No. 117, on December 31,2017 consolidated
/ group financial statements the amount should Par report as consolidated
common stock:
a. PI 00,000 c. PI 50,000
b. PI 12,500 d. P300,000
19. On January 1, 2013, Bristol,Company acquired 80 percent of Animation
Company's common stock for P280,000 cash. At that date, Animation rfeported
common stock outstanding of P200,000 and retained earnings of PI00,000, and
the fair value of the noncontrolling interest was P70,000. The book values and fair
values of Animation's assets and liabilities were equal, except for other intangible
assets which had a fair value P50,000 greater than book value and an 8-year
remaining life. Animation reported the following data for 2013 and 2020:

Year Net Income Comprehensive Income Dividends Paid


2013 P25,000 P30,000 P5.000
2020 35,000 45,000 10,000
Bristol reported separate net income from own operations of PI00,000 and paid
dividends of P30,000 for both the years.

What is the amount of consolidated comprehensive income reported for 2013?

a. PI 25,000 c. PI 18,750
b. PI 23,750 d. PI 30,000

120. Using the same information in No. 120, what is the amount of comprehensive
income attributable to the controlling interest for 2013?
a. PI 23,750 c. PI 19,000
b. PI 18,750 d. P104,000
121. Using the same information in No. 120, what is the amount of consolidated
comprehensive income reported for 2020?
a. P145,000 c. P138,750
b. P135,000 d. P128,750
122. Using the same information in No. 120, what is the amount of comprehensive
income attributable to the controlling interest for 2020?
a. PI 38,750 c. PI 28,750
b. P131,000 d. P135,000
123. On January 1, 2013, Post Company acquired an 80% investment in Stake
Company. The acquisition cost was equal to Post's equity in Stake's net assets at
that date, on January 1,2013, Post and Stake had retained earnings of.P500,000 and
P100,000, respectively. During 2013, Post had net income of P200,000, which
included its equity in Stake's earnings, and declared dividends of P50,000. Stake's
net income and dividends for 2013 amounted to P40,000 and P20,000, respectively.
There were no other intercompany transactioons between the parent and
subsidiary. On December 31, 2013, what should the consolidated retained earnings
be?
a. P650,000 c. P766,O00
b. P666,000 d. P770,000
119. On January 1, 2013, Bristol. Company acquired 80 percent of Animation
Company's common stock for P280,000 cash. At that date, Animation
rfeported common stock outstanding of P200,000 and retained earnings of
PI00,000, and the fair value of the noncontrolling interest was P70,000. The
book values and fair values of Animation's assets and liabilities were equal,
except for other intangible assets which had a fair value P50,000 greater than
book value and an 8-year remaining life. Animation reported the following data
for 2013 and 2020:

Year Net Income Comprehensive Income Dividends Paid


2013 P25,000 P30,000 P5.000
2020 35,000 45,000 10,000

Bristol reported separate net income from own operations of PI 00,000 and paid
dividends of P30,000 for both the years.

What is the amount of consolidated comprehensive income reported for 2013?

a. PI 25,000 c. PI 18,750
b. PI 23,750 d. PI 30,000

120. Using the same information in No. 120, what is the amount of comprehensive
income attributable to the controlling interest for 2013?
a. P123,750 c. PI 19,000
b. PI 18,750 d. PI 04,000
121. Using the same information in No. 120, what is the amount of consolidated
comprehensive income reported for 2020?
a. P145,000 c. P138,750
b. P135,O00 d. P128,750
122. Using the same information in No. 120, what is the amount of comprehensive
income attributable to the controlling interest for 2020?
a. P138,750 c. P128,750
b. P131,000 d. P135,O00
123. On January 1, 2013, Post Company acquired an 80% investment in Stake
Company. The acquisition cost was equal to Post's equity in Stake's net assets
at that date, on January 1,2013, Post and Stake had retained earnings of P.500,000
and PI00,000, respectively. During 2013, Post had net income of P200,000,
which included its equity in Stake's earnings, and declared dividends of
P50,000. Stake's net income and dividends for 2013 amounted to P40,000 and
P20,000, respectively. There were no other intercompany transactioons between
the parent and subsidiary. On December 31,2013, what should the consolidated
retained earnings be?
a. P650,000 c. P766,000
b. P666,000 d. P 770,000
124. On January 1, 2013, Wilhelm Corporation acquired 90 percent of Kaiser
Company's voting stock, at underlying book value. The fair value of the
noncontrolling interest was equal to 10 percent of the book value of Kaiser at
that date. Wilhelm uses the equity method in accounting for its ownership of
Kaiser. On December 31, 2020, the trial balances of the two companies are as
follows:
Debit Credit Debit Credit
Current assets P200,000 PI40,000
Depreciable assets 350,000 250,000
Investment in Kaiser Company 162,000
Depreciation expense 27,000 10,000
Other expenses 95,000 60,000
Dividends declared 20,000 10,000
Accumulated depreciation PI 18,000 P 80,000
Current liabilities 100,000 80,000
Long-term debt 100,000 50,000
Common stock 100,000 50,000
Retained earnings 150,000 100,000
Sales 250,000 110,000
Income from subsidiary 36,000
I
P854.000 P854.000 P470.000 P470.000

Based on the preceding information, what amount would be reported


retained earnings in the consolidated balance sheet prepared at December
31,2020?

a. P424,000 c. P294,000
b. P314,000 d. PI 50,000

125. On January 1, 2018, Piimsol Company acquired 100 percent of Shipping


Corporation's voting shares, at underlying book value. Piimsol uses the cost
method in accounting for its investment in Shipping. Shipping's retained earnings
was P75,000 on the date of acquisition. On December 31, 2018, the trial
balance data for the two companies are as follows:

Debit Credit Debit Credit


Current assets PI00,000 P75,000
Depreciable assets (net) 200,000 • 150,000
Investment in Shipping Company 125,000
Depreciation expense 20,000 15,000
Other expenses 60,000 45,000
Dividends declared 25,000 15,000
Current liabilities 40,000 25,000
Long-term debt 75,000 50,000
Common stock 100,000 50,000
Retained earnings 150,000 75,000
Sales 150,000 100.000
Dividends income 15,000
P530.000 P530.000 P300,000 P300.000
Based on the information provided, what amount of retained earnings will
be reported in the consolidated balance sheet prepared on December 31,
2018?

a. P310,000 c. P225,000
b. P235,000 d. P210,000

Intercompany Sales - Inventory

Items 126 through 128 are based on the following data:


Prid Corporation owns an 80% interest in Sed Corporation; and at December 31,
2017, Prid investment in Sed on a cost basis was equal to 80% of Sed's stockholders
equity. During 2017, Sed sold merchandise to Prid to PI00,000 at a gross profit to Sed of
P20,000. At December 31, 2018 half of this merchandise is included in Prid's inventory.
Separate incomes for Prid and Sed for 2018 are summarized as follows:
Prid Sed
Sales P500.000 P300,000
Cost of sales ( 250,000) ( 200,000)
Gross profit P250,000 PI00.000
Operating expenses ( 125,000) ( 40,000)
Separate incomes PI25,000 P 60,000

126. The Income from Sed for 2018 is:

a. P48,000 c. P8,000
b. 40,000 d. 0

127. The Consolidated/group cost of sales for 2018 is:

a. P460,000 c. P440,000
b. 450,000 d. 360,000
128. The Non-controlling interest in net income for 2018 is:

a. P60,000 c. PI 2,000
b. 48,000 d. 10,000

Items 129 through 132 are based on the following information:


The separate incomes (which do not include investment income) of Pell
Corporation and Sell Corporation, its 80% owned subsidiary, for 2018 were
determined as follows:

Pell Sell
Sales P400,000 P100,000
Less Cost of sales 200,000 60,000
Gross profit P200,000 P 40,000
Other expenses 100,000 30,000
Separate incomes PI00,000 P 10,000

During 2018 Pell sold merchandise that cost P20,000 to Sell for P40,000, and at
December 31, 2018 half of these inventory items remained unsold by Sell.

129. The Non-controlling interest in net income for 2018:


a. P 0 ,. c. P 8,000
b. 2,000 d. 10,000

130. The Consolidated sales for 2018:


a. P500.000 c. P460.000
b. 480,000 d. 400,000

131. The Consolidated cost of sales for 2018:


a. P230,000 c. P270,000
b. 248,000 d. 300,000

132. The Profit attributable to Equity Holders of Parent or CNI Contributable to


controlling Interests for 2018:
a. PI 08,000 c. P98,000
b. 100,000 d. 80,000
Items 133 through 135 are based on the following information:
Income statement information for the year 2018 for Perfect Corporation and its
60% owned subsidiary, Seven Corporation, is as follows:

Perfect Seven
Sales P900,000 P350.000
Cost of sales 400,000 250,000
Gross profit P500,000 P100,000
Operating expenses 250,000 50,000
Seven's net income P 50,000
Perfect's separate income P250,000

Intercompany sales for 2018 are upstream (from Seven to Perfect) and total
P100,000. Perfect's December 31,2017 and December 31,2018 inventories contain
unrealized profits of P5,000 and PI0,000, respectively.

133. The Consolidated sales for 2018:

a. P 900,000 c. PI, 190,000


b. 1,150,000 d. 1,250,000

134. The Consolidated cost of sales for 2018>

a. P545,O00 c. P555,000
b. 550,000 d. 560,000

135. The Profit attributable to Equity Holders of Parent or CNI Contributable to


Controlling Interests for 2018:

a. P277,000 c. P282,000
b. 280,000 d. 305,000

136. PP Corp. owns 80% of SS Inc.'s common stock. During 2018, PP.sold SS
P250,000 of inventory on the same terms as sales made to third parties. SS sold all
of the inventory purchased from PP in 2018. The following information pertains toSS
and PP's sales for 2018:

PP SS
Sales PI,000,000 P700,000
Cost of sales 400,000 350,000
P 600,000 P350.000
What amount should PP report as Cost of sales in its 2018 consolidated
income statement?

a. P750,000 c. P500,000
b. 680,000 d. 430,000
(AICPA)

137. Parry Corporation owns an 80% interest in Starry Corporation acquired


several years ago. Starry regularly sells merchandise to its parent at 125% of
Starry's cost. Gross profit data of Parry and Starry for the year 2018 are as
follows:

Parry Starry
Sales PI,000,000 P800,000
Cost of goods sold 800,000 640,000
Gross profit P 200,000 PI60,000

During 2018, Parry purchased inventory items from Starry at a transfer price of
P400,000. Parry's December 31,2017 and 2018 inventories included goods acquired
from Starry of P100,000 and P125,000, respectively. The Consolidated sales of Parry
Corporation and subsidiary for 2018 were:

a. PI,800,000 c. PI,400,000
b. 1,425,000 d. 1,240,000
(Adapted)

138. Using the same information in No. 137, the Unrealized profits in the year-
end 2017 and 2018 inventories were:

a. PI 00,000 and P125,000, respectively


b. P800.000 and P100,000, respectively
c. P20,000 and P25,000, respectively
d. PI 6,000 and P20,000, respectively
(Adapted)

139. Using the same information in No. 137, the Consolidated cost of goods
sold of Parry and subsidiary for 2018 was:

a. PI,024,000 c. PI,052,800
b. 1,045,000 d. 1.056,000
(Adapted)
140. • Power Co. is a manufacturer and Slack Co., its 100%-owned subsidiary, is a
retailer. The companies are vertically integrated. Thus, Slack purchases all
of its inventory from Power. On January 1,2018. Slack's inventory was P30,000.
For the year ended December 31,2018, its purchases were P150,000, and its
cost of sales was PI 66,500. Power's sales to Slack reflect a 50% markup on
cost. Slack then resells the goods to outside entitles at a 100% markup on
cost. At what amount should the intercompany inventory purchased from
Power be reported in the consolidated balance sheet at December 31,
2018?

a. P3,000 c. PI 3,500
b. 9,000 d. 46,000
(Adapted)

141. Bruce Company owns 80% of Lee Corp.'s common stock. During October
2018, Lee sold merchandise to Bruce for PI00,000. At December 31, 2018, one-
half of the merchandise remained in Bruce inventory. For 2018, gross profit
percentages were 30% for Bruce and 40% for Lee. The amount of unrealized
intercompany profit in ending inventory at December 31, 2018 that should be
eliminated in consolidation is:

a. P40,000 c. PI 6,000
b. 20,000 d. 15,000
(AICPA)

142. Sailing Company owns 100% of the capital stock of both Twill Corp. and
Webb Corp. Twill purchases merchandise inventory from Webb at 140% of Webb's
cost. During 2018, merchandise that cost Webb P40,000 was sold to Twill.
Twill sold all of this merchandise to unrelated customers for P81,200 during
2018. In preparing combined financial statements for 2018 Sailing's
bookkeeper disregarded the common ownership of Twill and Webb. By what
amount was unadjusted revenue overstatedtn the combined income
statements for 2018 and the amount that should be eliminated from cost of
goods sold in the combined income statement for 2018?

Overstated Unadjusted Cost of goods sold


Revenue to be eliminated
a. PI 6,000 PI 6,000
b. 40,000 40,000
c. 56,000 56,000
d. 81,200 56,000
(AICPA)
143. The Maroons Company holds a 70% interest in The Haena Company. At the
current year end Maroons holds inventory purchased from Haena for
P270,000 at cost plus 20%. The group's consolidated statement of financial position
has been drafted without any adjustments in relation to this holding of inventory.

What adjustments should be made to the draft consolidated statement of


financial position figures for non-controlling interest and retained earnings?

Non-controlling interest Retained earnings


a. No change Reduce by P45.000
b. No change Reduce by P54.000
c. Reduce by P16,200 Reduce by P37,800
d. Reduce by PI3,500 Reduce by P31,500

144. Rosas Corp. acquired a 70% interest in Camia Co. in 2017. For the year
ended December 31, 2017 and 2018, Camia Co. reported net income of
P160,000 and P180,000, respectively. During 2017, Camia sold merchandise to
Rosas Corp. for P20,000 at a profit of P4,000. The merchandise was later resold by
Rosas Corp. to outsider for P30,000 during 2018. For consolidation purposes, what is
the non-controlling interest's share of Camia's net income for 2017 and 2018,
respectively?

2017 2018
a. P46,800 P55.200
b. P48,000 P54,000
c. P49,000 P52,800
d. P53,200 P50.000
(PhilCPA)

145. On January 1,2013, Par Company purchased 80% of the outstanding shares
of Sub Company by paying P340.000, the Sub Company's common stock and
retained earnings on this date amounted to PI50,000 and P230,000
respectively. Also on this date, an equipment is undervalued by P20,000 with a
remaining life of 10 years.
On January 1, 2015, Sub Company had PI50,000 of capital stock and
P300,000 of retained earnings. Also on the same date. Par Company had
PI,000,000 of'capital stock and P70O000 of retained earnings.

During the year, Par Company sold merchandise to Sub for P60,000 and in turn,
purchased P40,000 from Sub Company. Inter-company sales of merchandise
were made at the following gross profit rates:

Sales made by parent 25% based on cost


Sales made by subsidiary 20% based on sales
On December 31,2015,30% of all inter-company sales remain in the ending
inventory of the purchasing affiliate.

The beginning inventory of Par Company includes P2,500 worth of


merchandise acquired from Sub Company on which Sub Company
reported a profit of PI,000. While, the beginning inventory of Sub also
includes P3,000 of merchandise acquired from Par Company at 35% mark¬ up.

Using cost method the following selected results of operations for 2015 were
as follows:

Par Company Sub Company


Dividends paid P 60,000 PIO.OOO
Net income from own operations PI 00,000 P30,000
Add: Dividend income 8,000
Net income P108,000 P30,000

The dividend income of Par Company for 2015 should be:

a. PI 8,330 c. P8,000
b. PI 0,000 d. P8,200

146. Using the same information in No. 145, the balance of Investment in Sub
Company as of December 31, 2015 should be:

a. P354,600 c. P35O.330
b. P351.960 d. P340,000

147. Using the same information in No. 145, the Non-controlling Interest in Net
Income for 2015, should be:

a. P6,280 c. P5,720
b. P6,120 d. P5,320

148. Using the same information in No. 145, the Profit Attributable to Equity
Holders of Parent/Controlling Interest in Net Income for 2015 should be:

a. PI 22,600 c. PI 18,570
b. PI 18,730 d. PI 18,330
149. Using the same information in No. 145, the Consolidated Net Income /
Group Net Income for 2015 should be:

a. PI 24,050 c. PI 18,570
b. PI 22,600 d. PI 18,330

150. Using the same information in No. 145, the parent's portion of consolidated
(for controlling interest / equity holders of parent) retained earnings on
December 31,2015:

a. P700,000 c. P753,600
b. P752,000 d. P809.680

151. Using the same information in No. 145, the consolidated retained earnings
on December 31,2015:

a. P700.000 c. P753.600
b. P752.000 d. P809.680

152. Using the same information in No. 145, the stockholders' equity of subsidiary
on December 31, 2015 should be:

a. P450.000 c. P481.600
b. P470,O00 d. P484,000

153. Using the same information in No. 145, the Non-controlling Interest (in Net
Assets) on December 31,2015 using proportionate basis (or partial goodwill
approach) should be:

a. P97,120 c. P96,320
b. P96,920 d. P73,520

154. Using the same information in No. 145, the Non-controlling interest (in Net
Assets) on December 31, 2015 using full fair value basis (or full-goodwill
approach) should be:

a. P101.320 c. P96.320
b. P 96,920 d. P73.520
155. Using the same information in No. 145, the consolidated stockholders' equity
on December 31, 2015 using proportionate basis (or partial goodwill
approach):

a. PI,911,000 c. PI,905,920
b. PI,906,000 c. PI, 740,000

156. Using the same information in No. 145, the consolidated stockholders' equity
on December 31,2015 using full fair value basis (or full-goodwill approach)
should be:

a. PI,911,000 c. PI,905,920
b. PI,906,000 d. PI,740,000

Intercompany Sales - Depreciable and Non-depreciable Assets

Items 157 through 159 are based on the following information:


Income information for 2018 taken from the separate company financial
statements of Peras Corporation and its 75% owned subsidiary. Sky Corporation is
presented as follows:

Peras Sky
Sales PI,000,000 P46O000
Gain on sale of building 20,000
Dividend income 75,000
Cost of goods sold ( 500,000) ( 260,000)
Depreciation expense ( 100,000) ( 60,000)
Other expenses ( 200,000) ( 40,000)
Net income P 295,000 PI00,000

Peras gain on sale of building relates to a building with a book value of P40,000 and a
10-year remaining useful life that was sold to Sky for P60,000 on January 1, 2018.

157. At what amount will the gain on sale of building appear on the
consolidated/group income statement of Peras and Sky for the year 2018
should be:

a. Zero c. PI 5,000
b. P5,000 d. 20,000
158. The Consolidated/group depreciation expense for'2018 should be:

a. PI 58,000 c. PI 62,000
b. 160,000 d. 180,000

159. The Profit Attributable to Equity Holders of Parent or CNI Contributable to


Controlling Interests for 2018 should be:

a. P295,000 c. P275,000
b. 277,000 d. 220,000

Items 160 through 162 are based on the following information.


Saul is a 90%-owned subsidiary of Paul Corporation, acquired at book value several
years ago. Comparative separate company income statements for these affiliated
corporations for 2018 are as follows:

Paul Saul
Corporation Corporation
Sales PI,500,000 P700,000
Dividend income 108,000
Gain on building 30,000 -
Income credits PI,638,000 P700,000
Cost of sales PI,000,000 P400,000
Operating expenses 300,000 150,000
Income debits PI,300,000 P550,000
Net income P 338,000 PI50,000

On January 5, 2018 Paul sold a building with a 10-year remaining useful life to
Saul as a gain of P30,000. Saul paid dividends of PI20,000 during 2018.

160. The Non-controlling interest in net income for 2018:

a. PI 2,000 c. PI 5,000
b. .12,300 d. 15,300

161. The Profit Attributable to Equity Holders of Parent or CNI Attributable to


Controlling Interests for 2018:

a. P342,000 c. P338,000
b. 340,700 d. 335,000
162. The Consolidated/group net income for 2018 should be:

a. P338,000 c. P380,000
b. 353,000 d. 443,000

163. Silver Corporation is a 90% owned subsidiary of Proto Corporation acquired


several years ago at book value equal to fair value. For the years 2017 and
2018, Proto and Silver report the following:

2017 2018
Proto's separate income P300,000 P400,000
Silver's net income 80,000 60,000

The only intercompany transaction between Proto and Silver during 2017 and
2018 was the January 1, 2017 sale of land. The land had a book value of
P20,000 and was sold intercompany for P30,000, its appraised value at the time
of sale.

If the land was sold by Proto to Silver (downstream sales) and that Silver still owns
the land at December 31, 2018, compute the Profit Attributable to Equity
Holders of Parent for 2017 and 2018:

2017 2018 2017 2018


a. P363,000 P454,000 c. P372,000 P460,000
b. 362,000 454,000 d. 362,000 460,000
(Adapted)

164. Using the same information in No. 163, the Consolidated/group net income
for 2017 and 2018:

2017 2018 2017 2018


a. P362,000 P454,000 c. P370,O00 P460,0O0
b. 380,000 460,000 d. 372,000 460,000

165. Using the same information in No. 163, except that the land was sold by
Silver to Proto (upstream sales) and Proto still owns the land at December 31,
2008, compute the Profit Attributable to Equity Holders of Parent or CNI
Attributable to Controlling Interests for 2017 and 2018:

2017 2018 2017 2018


a. P363,000 P454,000 c. P370,000 P460,O00
b. 362,000 454,000 d. 363,000 460,000
(Adapted)
166. Using the same information in No. 165, the Consolidated/group net income
for 2017 and 2018:

2017 2018 . 2017 2018


a. 362,000 P454,000 c. P370,000 P460,000
b. 380,000 460,000 d. 372,000 460,000

167. PP Corp. owns 100% of SS Corp.'s common stock. On January 2, 2017, PP


sold to SS for P40,000 machinery with a carrying amount of P30,000. SS is
depreciating the acquired machinery over a five year life by the straightline
method. The net adjustments to compute 2017 and 2018 Profit
Attributable to Equity Holders of Parent or CNI Attributable to Controlling
Interests before income tax would be an increase (decrease) of:

2017 2018
a. P( 8,000) P2,000
b. ( 8,000) 0
c. (10,000) 2,000
d. (10,000) 0
(AICPA)

168. The Snipes Company owns 65% of The Genie Company. On the last day of
the accounting period Genie sold to Snipes a non-current asset for P200,000. The
asset originally cost p500,000 and at the end Of the reporting period its carrying
amount in genie's books was PI 60,000. The group's consolidated statement of
financial position has been drafted without any adjustments in relation to this
non-current asset.

What adjustments should be made to the consolidated statement of


financial position figures for non-current assets and retained earnings?

Non-current assets Retained earnings


a. Increase by P300,000 Increase by P195,000
b. Reduce by P40,000 Reduce by P26,000
'c. Reduce by P40,000 Reduce by P40,000
d. Increase by P300,000 Increase by P300,000
169. The Lakers Company owns 75% of The Viking Company. On December 31,
2018, the last day of the accounting period, Vikings sold to Lakers a
noncurrent asset for P200,000. The asset's original cost was P500,000 and on
December 31, 2018 its carrying amount in Viking's books was PI 60,000. The
group's consolidated statement of financial position has been drafted
without any adjustments in relation to this non-current asset.

What adjustments should be made to the consolidated statement of


financial position figures for retained earnings and non-controlling interest?

Retained earnings Non-controlling interest


a. I ncrease by P225,000 Increase by P75,000
b. Increase by P300,000 No change
c. Reduce by P30,000 Reduce by P10,000
d. Reduce by P40,000 No change

170. The Virgil Company owns 65% of The Migu Company. On December 31,
2018, the last day of the accounting period, Virgil sold to Migu a noncurrent asset
for PI ,000. The asset's original cost was p2,500 and on December 31, 2018 its
carrying amount in Virgil's books was P800. The group's consolidated statement of
financial position has been drafted without any adjustments in relation to this
non-current asset.

What adjustments should be made tot he consolidatred statement of


financial position figures for non-current assets and non-controlling interest?

Non-current assets Non-controlling interest


a. Increase by PI ,500 Increase by P525
b. Reduce by P200 No change
c. Reduce by P200 Reduce by P70
d. Increase by PI,500 No change

171. The Roel Company acquired equipment on January 1, 2009 at a cost of


P800,000, depreciating it over 8 years with a nil residual value. On January 1,
2018. The Muldon Company acquired 100% of Roel and estimated the fair value
of the equipment at'P460,000, with a remaining life of 5 years. This fair value was
not incorporated info Roel's books and the depreciation expense continued
to be calculated by reference to original cost.
What adjustments should be made to the depreciation expense for the year
and the statement of financial position carrying amount in preparing the
consolidated financial statements for the year ended December 31, 2013?

Depreciation expense Carrying amount


a. Increase by P8,000 » Increase by P24,000
b. Increase by P8,000 Decrease by P24,000
c. Decrease by P8,000 Increase by P24,000
d. Decrease by P8,000 Decrease by P24,000

172. On January 1,2018, Poe Corp. sold machine for P900,000 to Saxe Corp., its
wholly owned subsidiary. Poe paid P1,100,000 for this machine, which had
accumulated depreciation of P250,000. Poe estimated a PI00,000 salvage value
and depreciated the machine on the straight-line method over 20 years, a
policy which Saxe continued. In Poe's December 31, 2018, consolidated
balance sheet, this machine should be included in cost and accumulated
depreciation as:

Cost Accumulated depreciation


a. PI, 100,000 P300,000
b. 1,100,000 290,000
c. 900,000 40,000
d. 850,000 42,500
(AICPA)

173. On January 1,2018, Jan Company purchased 90% equity of Jo Company. On


January 3,2018, Jo sold equipment (with original cost of P750,OO0 and carrying
cost of P375,000) to Jan for P540,000. The equipment have a remaining life of three
(3) years and was depreciated using the straight-line method by both
companies. In Jan consolidated balance sheet as of December 31,2018, the cost,
accumulated depreciation and book value should be reported at:

Cost Accumulated Depreciation Net Book Value


a. P750,000 P500,000 P375,000
b. 375,000 375,000 -0-
c. 750,000 750,000 -0-
d. 750,000 500,000 250,000
(Adapted)
174. As January 1,2018, Johnson Corporation sold equipment with a three-year
remaining useful life and a book value of P10,000 to its 70%-owned subsidiary for a
price of PI 1,500. In the consolidation working papers for the year ended
December 31,2018, the elimination entry concerning this transaction will include:

a. A debit to equipment for PI ,500.


b. A debit to gain on equipment for PI ,500.
c. A credit to depreciation expense for P1,500.
d. A debit to gain on equipment sale for PI ,000.
(Adapted)

175. As January 1, 2018, Par Corp. sold a warehouse with a book value of P80,000
and a 20-year remaining useful life to its wholy-owned subsidiary, Strata
Corporation, for PI 20,000. Both Par and Strata use the straight-line depreciation
method. On December 31,2018, the separate company financial statements
contained the following balances connected with the warehouse:

Par Strata
Gain on sale of warehouse P40,000
Depreciation expense P 6,000
Warehouse 120,000
Accumulated depreciation 6,000

A working paper entry to consolidate the financial statements of Par and


Strata on December 31, 2018 will include:
a. A debit to gain on sale of warehouse for P38,000.
b. A debit to gain on sale of warehouse for P40,000.
c. A debit to accumulated depreciation for P2,000.
d. A credit to depreciation expense for P6,000
(Adapted)

176. On January 1,2017, Pure Company purchased 80 percent of the outstanding


shares of Sure Company at a cost of PI,000,000. On that date, Sure
Company had P400,000 of capital stock and P600,000 of retained earnings.
On July 1, 2017, Sure Company sold an equipment with a book value of
P60,000 to Pure Company for P80,000.

For 2017 and 2018, the results of their operations are:

2017 2018
Pure Co. Sure Co. Pure Co. Sure Co.
Net income from own operations....... P400,000 P200,000 P300,000 PI50,000
Dividends paid 100,000 50,000 80,000 20,000
The intercompany gain is included in the net income of Sure Company.
The equipment sold is expected to have a useful life of five years from
the date of sale.

The Non-controlling interests on December 31:


20J1 2018 2017 2018
a. P226,400 P256,800 c. P226,O00 P252,400
b. 226,400 253,200 d. 230,000 256,000
(Adapted)

177. Scroll, Inc. a wholly owned subsidiary of Pirn, Inc. began operations on
January 1, 2018. The following information is from the condensed 2018
income statements of Pirn and Scroll:
Pirn Scroll
Sales to Scroll P100.000 P
Sales to others 400,000 300,000
P500,000 P300,000
Cost of goods sold:
Acquired from Pirn - 80,000
Acquired from others 350,000 190,000
Gross profit P150,000 P 30,000
Depreciation 40,000 10,000
Other expenses 60,000 15,000
Income from operations P 50,000 P 5,000
Gain on sale of equipment to
Scroll 12,000
Income before income taxes P 62,000 P 5,000

Additional Information:
* Sales by Pirn to Scroll are made on the same terms as those made
to third parties.
* Equipment purchased by Scroll from Pirn for P36,000 on January 1,
2018 is depreciated using the straight-line method over four-years.
For purposes of consolidation on December 31, 2018, what amount of
intercompany profit that should be eliminated from Scroll's inventory in
the consolidated financial statements?
a. P6,TJ00 c. P2O000
b. PI 0,000 d. P30.000
(AICPA)

178. Using the same information in No. 177, the amount of depreciation expense
in the consolidated F/S?
a. P44,000 c. P50,000
b. P51.000 d. P53,000
179. Schoenfeld Corporation is an 80% owned subsidiary of Pax Corporation. In
2017, Schoenfeld sold land net cost P15,000 to Pax for P25,000. Pax held the land
for eight years before reselling it in 2018 to Eddie Company, an unrelated entity, for
P55,000. The consolidated income statement for Pax and its subsidiary in
2018, Schoenfeld, will show a gain on the sale of land of:

a. P40,000 c. P30,000
b. 32,000 d. 24,000
(Adapted)

180. Justings Co. owned 80% of Evana Corp. During 2017, Justings sold to Evana
land with a book value of P48,000. The selling price was P70,000. In its
accounting records, Justings should:

a. Not recognize a gain on the sale of the land since it was made
to a related party
b. Recognize a gain of PI 7,600
c. Defer recognition of the gain until Evana sells the land to a third
party
d. Recognize a gain of P22,000

181. On January 1,2013, P Company purchased 80 percent of the outstanding


shares of S Company by paying P700,000. On that date, S Company had
P300,000 capital stock and P500,000 of retained earnings. An undervalued asset
attributable to building amounting to P75,000 with a remaining life of 25 years.
All other assets and liabilities of S Company had book value approximated their
fair market value.

On January 1, 2020, P's common stock and retained earnings amounted to


PI,000,000 and P800,000, respectively, while S Company's retained earnings is
P600,000.

The 2020 net income and dividends using cost (or initial value) model was as
follows:

Net Income Dividends


• P Company P340,000 P100,000
S Company P 150,000 P50,000

On April 1,2020, S Company sold equipment with a book value of P30,000 to P


Company for P60,000. The gain on the sale is included in the net income of
S Company indicated above. The equipment is expected to have a remaining
useful life of five years fromt he date of the sale.
On September 30, 2020, P Company sold machinery with a book value of
P40.000 to S Company for P75,000. The gain on the sale is also included in the
net income of P Company indicated above. The machinery is expected to last
for ten (10) years from the date of sale.

The investment in Subsidiary account on December 31,2020:

a. P748,500 c. P721,600
b. P725,000 d. P700,000

182. Using the same information in No. 181, the Dividend income/investment
income for 2020:

a. P88,500 c. P61,600
b. P65,000 d. P40,000

183. Using the same information in No. 181, the Non-controlling interest in Net
Income for 2020:

a. P30,000 c. P24,900
b. P25,500 d. P24.300

184. Using the same information in No. 181, the Profit attributable to equity
holders of parent (Parent's Interest/Controlling Interest in Profit) for 2020:

a. P356,500 c. P363,075
b. P362,200 d. P386,500

185. Using the same information in No. 181, the Consolidated/Group Net Income
for 2020:

a. P356,500 c. P363,075
b. P362,200 d. P387,375

186. Using the same information in No. 181, the parent's portion of consolidated
(or controlling interest/equity holders of parent) retained earnings on
December 31,2020:

a. PI,040,000 c. PI, 123,075


b. PI,063,075 d. PI,140,675
187. Using the same information in No. 181, the consolidated retained earnings
on December 31,2020:

a. PI,040,000 c. PI, 123,075


b. PI,063,075 d. PI, 140,675

188. Using the same information in No. 181, the non-controlling interest on
December 31,2020:

a. P208,700 c. PI 74,900
b. P189,3O0 d. P173.100

189. Using the same information in No. 181, the non-controlling interest (in net
assets) on December 31,2015, assuming that the net income and dividends
of subsidiary amounted to P200,000 and P70,000, respectively:

a. P208,000 c. P235,300
b. P209,200 d. P222,4000

190. Using the same information in Nos. 181, 186 and 187, compute the
stockholders' equity of subsidiary on December 31:

2020 2015 2020 2015


a. PI,000,000 PI,000,000 c. PI,069,000
b. PI,000,000 PI, 130,000 d. PI,043,500 PI, 176,500

191. Using the same information in No. 181, the consolidated stockholders' equity
on December 31,2020:

a. P2,040,000 c. P2,358,375
b. P2,349,375 d. P2,375,975

Intercompany Accounts

192. Par Corp. owns 60% of Sub Corp.'s outstanding capital stock. On May 1,
2017, Par advanced Sub P70,000 in cash, which was still outstanding at
December 31,2018. What portion of this advance should be eliminated in
the preparation of the December 31,2018 consolidated balance sheet?

a. P70,000 , c. P28,000
b. 42,000 d. 0
(AICPA)
193. During 2018, Pard Corp. sold goods to its 80% owned subsidiary, Seed Corp.
At December 31,2018, one-half (112) of these goods were included in Seed's ending
inventory. Reported 2018 selling expenses were PI,100,000 and P400,000 for
Pard and Seed, respectively. Pard's selling expenses included P50,000 in freight-
out costs for goods sold to Seed. What amount of selling expenses should be
reported in Pard's consolidated income statement?
a. PI,500,000 c. PI, 475,000
b. 1,480,000 d. 1,450,000
(AICPA)

194. At December 31,2018, Grey, Inc. owned 90% of Winn Corp., a consolidated
subsidiary, and 20% of Carr Corp., an investee over which Grey cannot
exercise significant influence. On the same date, Grey had receivables of
P300,000 from Winn and P200,000 from Carr. In its December 31, 2018
consolidated balance sheet, Grey should report accounts receivable from
affiliates of:
a. P500,000 c. P230,000
b. 340,000 d. 200,000
(AICPA)
195. Dean, Inc. owns 100% of Roy Corporation, a consolidated subsidiary, and
80% of Wall, Inc., an unconsolidated subsidiary at 12/31. On the same date, Dean
has receivables of P200,000 from Roy and PI 75,000 from Wall. In its 12/31
consolidated balance sheet, Dean should report accounts receivable from investees
at
a. P 0 c. PI 75,000
b. 35,000 d. 235,000
(AICPA)
196. Wright Corp. has several subsidiaries that are included in its consolidated
financial statements. In its December 31, 2017, trial balance, Wright had the
following intercompany balances before eliminations:
Debit Credit
Current receivable due from Main Co P32,000
Noncurrent receivable from Main 114,000
Cash advance to Corn Corp 6,000
Cash advance from King Co P15.000
Intercompany payable to King 101,000
In its December 31,2017, consolidated balance sheet, what amount should
Wright report as intercompany receivables?
a. PI 52,000 c. P36,000
b. 146,000 d. 0
(AICPA)

197. The Carly Company owns 75% of The Halley Company. The following figures
are from their separate financial statements:

Carly: Trade receivables P1,040,000, including P30,000 due from Halley.


Halley: Trade receivables P215,000, including P40,000 due from Carly.

What figure should appear for trade receivables in Carly's consolidated


statement of financial position?

a. PI,215,000 c. PI,255,000
b. PI,225,000 d. PI, 185,000

198. Cobb Company's current receivables from affiliated companies at


December 31, 2017 are (1) a P75,000 cash advance to Hill Corporation
(Cobb owns 30% of the voting stock of Hill and accounts for the investment by
the equity method), (2) a receivable of P260,000 from Vick Corporation for
administrative and selling services (Vick is 100%-owned by Cobb and is
included in Cobb's consolidated financial statements), and (3) a receivable of
P200,000 from Ward Corporation for merchandise sales on credit (Ward is a
90%-owned, unconsolidated subsidiary of Cobb accounted for by the equity
method). In the current assets section of its December 31, 2017
consolidated balance sheet, Cob should report accounts receivable from
investees in the amount of:

a. PI 80,000 c. P275,OO0
b. 255,000 d. 535,000
(AICPA)

199. Pero Corporation owns a 70% interest in Sweet Corporation, acquired


several years ago at book value. On December 31, 2017, Sweet mailed a
check for PI0,000 to Pero in part payment of a P20.000 account with Pero.
Pero had not received the check when its books were closed on December
31. Pero Corporation had accounts receivable of P150,000 (including the
P20,000 from Sweet) and Sweet had accounts receivable at P22O000 at
year-end. In the consolidated balance sheet of Pero Corporation and
Subsidiary at December 31,2017 accounts receivable will be shown in the
amount of:

a. P370,000 c. P350,000
b. 360,000 d. 304,000
(Adapted)
200. Clark Co. had the following transactions with affiliated parties during 2017:

Sales of P60,000 to Dean, Inc., with P20,000 gross profit. Dean had
P15,000 of this inventory on hand at year end. Clark owns a 15% interest in
Dean and does not exert significant influence.
• Purchases of raw materials totaling P240,000 from Kent Corp., a wholly-
owned subsidiary. Kent's gross profit on the sale was P48,000. Clark had
P60,000 of this inventory remaining on December 31, 2017.

Before eliminating entries, Clark had consolidated current assets of P320,000.


What amount should Clark report in its December 31, 2017, consolidated
balance sheet for current assets?

a. P320,000 c. P308,000
b. 317,000 d. 303,000
(AICPA)

Appendix for SMEs - Consolidated and Separate Financial Statements

Some of the key differences between Full PFRSs (F-PFRS)a dn PFRS for SMEs (SMEs) that
affects computational aspect of consolidated and separate financial
statements are as follows:

F-PFRS SMEs

1. Accounting for invest¬ Either: Either:


ment in subsidiaries, (a) at cost, or (a) at cost less impair¬
jointly controlled enti¬ ment, or
ties and associates (b) in accordance with (b) at fair value with
IAS (PAS) 39 changes in fair value
recognized in P&L

2. Non-controlling Inter¬ NCI can be measured NCI are stated at the non-
ests (NCI) in the using either: controlling interest portion
acquiree) 1. Fair value of NCI (full of the fair value of the net
goodwill); or assets of the entity ac¬
2. Proportionate interest quired (partial goodwill)
in the fair value of net
identifiable assets of
the entity acquired
(partial goodwill)

You might also like